U World ER

¡Supera tus tareas y exámenes ahora con Quizwiz!

Ovarian torsion Risk factors

. Ovarian mass Women of reproductive age Infertility treatment with ovulation induction

A 5 - year - old girl is brought in after a seizure at home . Approximately an hour ago , the patient collapsed at the dinner table and developed full body shaking and urinary incontinence . Her mother says the patient has had a mild headache and fatigue for the past few days . On examination , the patient is sleepy and difficult to rouse . Vital signs are normal . Cardiopulmonary examination is unremarkable . CT scan of the head reveals a ring - enhancing lesion with surrounding edema . Which of the following historical details is most relevant in establishing this patient's diagnosis ? A. Family history of seizures B. History of diarrheal illness C. Maternal history of perinatal infection D. Recent history of head trauma E . Recent history of otitis media

. Recent history of otitis media This patient likely has a patellar dislocation , which usually occurs after quick , lateral movements on a flexed knee and is most common in young athletes ( eg , soccer or ice hockey players , gymnasts ) , dancers , and military trainees . Patients may feel the knee giving way , followed by a popping noise and severe pain . The quadriceps muscles normally exert a lateral force on the patella , especially during quadriceps contraction ; in cases of patellar dislocation , lateral displacement with associated tear of the medial patellofemoral ligament ( which normally provides an opposing medial force ) is most common . Examination shows a flexed knee with reduced range of motion and lateral displacement of the patella ( seen as a large , immobile deformity in this patient ) out of the trochlea , which may be palpable as a depression at the anterior knee . Other findings can include hemarthrosis and tenderness along the medial patella , which reflect a tear of the medial patellofemoral ligament . X - rays may not be needed for diagnosis in clinically apparent dislocations but are performed after reduction to rule out additional injuries . Choice A ) Anterior cruciate ligament ( ACL ) tears are typically due to rapid deceleration or pivoting at the knee with the foot planted . Acute hemarthrosis is common in this injury and may limit extension of the knee , but the patella remains seated in the trochlea . Choice B ) Medial collateral ligament ( MCL ) tears are caused by severe valgus stress ( eg , blow to the lateral knee ) or a twisting injury . the phase , ligamentous laxity may be masked by swelling and muscle spasm , range of moti s preserved , and the patella would not be displaced . Choice C ) Meniscal tears often occur from pivoting on a flexed knee while the foot is planted . Exa

Ovarian torsion Clinical presentation

. Sudden - onset unilateral pelvic pain • Nausea & vomiting • Palpable adnexal mass

Myasthenia Gravis Treatmetn

1) ACETYLCHOLINESTERASE INHIBITORS NEOSTIGMINE or PYRIDOSTIGMINE -STOPS BREAKDOWN of ACETYLCHOLINE -INCREASES CONCENTRATION of ACETY LCHOLINE 2)IMMUNOSUPPRESSIVE DRUGS ( PREDNISONE ) -REDUCE PRODUCTION OF AUTOANTIBODIES 3)SURGICAL REMOVAL of THYMUS -REDUCES MUSCLE WEAKNESS

A 73 - year - old woman is brought to the emergency department due to a 10 - day history of abdominal pain and loss of appetite . Frequent , loose stools and loss of appetite have been present since the pain began . Temperature is 37.8 C ( 100 F ) and pulse is 98 / min . Abdominal examination is significant for tenderness on deep palpation of the left lower quadrant , along with an appreciable mass . There is no inguinal lymphadenopathy . Leukocyte count is 15,000 / mm³ . Which of the following is the most likely diagnosis in this patient ? A. Acute diverticulitis B. Colon cancer C. Incarcerated direct inguinal hernia D. Ischemic colitis E. Ulcerative colitis

A. Acute diverticulitis

A 32 - year - old woman presents with double vision and slurred speech that started several hours ago . Objects appear blurry , which she has never experienced . The patient ate home - preserved fish yesterday at a friend's house . Temperature is 36.6 C ( 97.9 F ) , blood pressure is 132/73 mm Hg , and pulse is 59 / min . On examination , the patient is alert , awake , and fully oriented . Speech is fluid but slurred . The oral mucosa is dry . The pupils are dilated with sluggish reactivity to light . The patient has mild bilateral ptosis and facial muscle weakness . Neck muscles are weak . Muscle strength is 4/5 in both upper extremities . Strength in the lower extremities is normal . Sensation is intact throughout . Which of the following is the best next step in management ? A. Administer antitoxin B. Administer corticosteroids C. Measure serum acetylcholine receptor antibodies D. Obtain lumbar puncture E. O

A. Administer antitoxin This patient has rapidly progressive neurologic deficits that include the following : • Double vision , impaired pupillary constriction , and ptosis • Bulbar symptoms ( eg , dysarthria ) , impaired salivary secretion ( eg , dry mouth ) • Symmetric , descending muscle weakness ( face , arms , relative sparing of legs ) Sensation is intact , and there is a lack of systemic inflammatory symptoms ( eg , afebrile , no tachycardia , normal mental status ) . This presentation is suspicious for foodborne botulism . Improperly canned foods and cured fish ( eg , alkaline foods ) provide the appropriate environment for spore germination of Clostridium botulinum , which produces a toxin that inhibits presynaptic acetylcholine release at the neuromuscular junction . This leads to rapidly progressive , symmetric , descending weakness ( limb and / or bulbar ) . Identification of risk factors ( eg , recent ingestion of home - canned food ) increases suspicion . Serum toxin assay confirms the diagnosis , but treatment with antitoxin ( preformed antibodies that inactivate Clostridium toxin ) should not be delayed if there is high clinical suspicion . Intubation and mechanical ventilation may be necessary in patients with respiratory failure . ( Choices B and E ) Corticosteroids are used to treat acute flares of multiple sclerosis ( MS ) , which can present with diplopia due to cranial nerve deficits ( eg , abducens nerve [ CN VI ] ) or lesions involving the medial longitudinal fasciculus ( coordinates gaze ) . However , MS presents with neurologic deficits disseminated in time and space . MRI of the brain and spine can often confirm the diagnosis by revealing lesions in different places and / or stages ( ie , time and space . This patient's symmetric motor deficits and eye findings that include impaired p

An 83 - year - old man is seen after he experienced sudden - onset dizziness . He has no associated chest pain . The patient has a history of 3 - vessel atherosclerotic coronary artery disease and prior myocardial infarction , for which he underwent coronary artery bypass surgery . On physical examination , the patient appears uncomfortable but is interactive and speaking in full sentences with no respiratory distress . Blood pressure is 100/60 mm Hg and pulse is 140 / min . Pulse oximetry shows 95 % on room air . Cardiac examination is notable for tachycardia with no audible murmurs . ECG rhythm strip is shown above . Which of the following medications is most likely to improve this patient's condition ? A. Amiodarone B. Atropine C. Digoxin D. Metoprolol E. Verapamil

A. Amiodarone

A 60 - year - old man is brought to the emergency department after passing out at home . The patient was watching television while seated in an armchair when he suddenly passed out with no warning . According to his wife , he was unconscious for 1-2 minutes . The patient has never had a similar episode . The patient had a myocardial infarction 6 months ago and spent 5 days in the hospital . He now appears comfortable and has no symptoms . Blood pressure is 130/85 mm Hg and pulse is 80 / min with frequent ectopic beats . Physical examination reveals a 2/6 holosystolic apical murmur radiating to the axilla . Which of the following is the most likely cause of this patient's event ? A. Arrhythmia B. Autonomic dysfunction C. Postural hypotension D. Seizure E. Vasovagal reaction

A. Arrhythmia Syncope is a transient loss of consciousness accompanied by a loss of postural tone with a spontaneous return to baseline neurologic function . It occurs due to global hypoperfusion of the brain and can have many causes . In many patients ( eg , those with hypovolemia or vasovagal syncope ) , syncope is benign and self - limiting ; however , it can be the initial manifestation of a life threatening disease process ( eg , cardiogenic syncope ) . This patient's syncope is most likely due to a cardiac arrhythmia . Findings consistent with an arrhythmic etiology in this patient include the following : Sudden onset with lack of prodromal symptoms Occurrence while the patient was at rest and seated ( ie , no change in position ) Evidence of underlying structural heart disease ( eg , prior myocardial infarction , murmur of mitral regurgitation ) Presence of frequent ectopic beats The most common arrhythmia causing syncope is ventricular tachycardia , which is potentially fatal . Therefore , this patient should be admitted to the hospital for continuous ECG monitoring and further cardiac workup ( eg , echocardiography Choice B ) Autonomic dysfunction resulting from degenerative changes to autonomic ganglia or CNS nuclei can result in syncope . However , it is usually seen in patients with neurodegenerative disease ( eg , Parkinson disease ) , and syncope typically occurs on standing due to an inadequate compensatory sympathetic response ( eg vasoconstriction ) . ( Choice C ) Orthostatic ( postural ) hypotension refers to a drop in systolic blood pressure > 20 mm Hg or diastolic blood pressure > 10 mm Hg within 2-5 minutes of standing from a supine position . It is a common cause of syncope but is unlikely in this patient who was sitting in an armchair when the syncopal episode occurred . ( Choice D ) Seizures c

A 32 - year - old woman is evaluated due to slurred speech and difficulty walking . The patient has also been uncharacteristically drowsy for the past several hours . Temperature is 37 C ( 98.6 F ) , blood pressure is 110/70 mm Hg , pulse is 76 / min , and respirations are 12 / min . The patient is lethargic and falls asleep during the examination . Pupils are 3 mm and reactive to light . The abdomen is soft and nontender ; bowel sounds are normal . Limited neurologic examination shows 2+ deep tendon reflexes in all 4 extremities , and strength is normal throughout . No nystagmus or hand tremor is present . Blood glucose is 130 mg / dL . Which of the following is the most likely cause of this patient's symptoms ? A. Benzodiazepine overdose B. Lithium toxicity C. Opioid intoxication D. Phenytoin toxicity E. Serotonin syndrome

A. Benzodiazepine overdose Benzodiazepines may be used to manage bipolar disorder , insomnia , seizures , and alcohol withdrawal ; however , the risks of overdose and physical dependence / withdrawal associated with benzodiazepines make temporary use more desirable than chronic administration whenever possible . This patient's slurred speech , ataxia ( ie , difficulty walking ) , and CNS depression ( eg , lethargy , drowsiness ) are consistent with a benzodiazepine overdose . Benzodiazepines are sedative - hypnotics with stereotypical features of intoxication : • Mild to moderate : CNS depression , ataxia , slurred speech ( as seen in this patient • Severe : respiratory depression / arrest , hypotension , hypothermia , coma CNS depression is the most common symptom in benzodiazepine overdose . Respiratory depression may also occur ; however , this effect generally happens after large overdoses or rapid intravenous administration of short - acting formulations ( eg , midazolam ) . Additionally , coingestion with other CNS depressants ( eg , opioids , alcohol ) greatly increases the risk for respiratory depression and death . ( Choice B ) Lithium toxicity can present with ataxia , lethargy , and slurred speech . However , gastrointestinal distress ( eg , nausea , vomiting , diarrhea ) , nystagmus , tremor , and / or hyperreflexia would likely occur as well . Choice C ) Opioid intoxication is likely to include sedation , ataxia , and slurred speech ; however , respiratory depression and miosis ( ie , pinpoint pupils ) are expected as well . This patient's normal vital signs ( eg , respiratory rate 12 / min ) , bowel sounds , and pupillary reaction ( reactive to light ) and size ( 3 mm ) are not expected in opioid overdose . ( Choice D ) Ataxia , confusion , and CNS depression are likely in phenytoin overdose ; howeve

A 64 - year - old man comes to the emergency department due to worsening left lower extremity pain , swelling , and redness over the past 3 days . The patient has had no trauma . Temperature is 38 C ( 100.4 F ) and BMI is 35 kg / m² . Physical examination shows diffuse erythema extending up to the left midcalf with an indistinct border . There is increased warmth , tenderness , and edema of the left leg . No areas of fluctuation or purulent exudate are present , but the interdigital skin of the feet is macerated and fissured . Leukocyte count is 15,000 / mm³ . Infection with which of the following organisms is most likely responsible for this patient's current symptoms ? A. Beta - hemolytic streptococci B. Gandida albicans C. Clostridium perfringens D. Pseudomonas aeruginosa E. Varicella - zoster virus

A. Beta - hemolytic streptococci This patient with skin findings ( eg , urticarial rash ) , respiratory difficulty ( eg , wheezing ) , and hypotension has anaphylaxis , likely caused by antibiotic exposure . Diagnosing anaphylaxis can be challenging if symptoms are initially attributed to a preexisting medical condition ( eg , asthma ) , as with this patient . Epinephrine is the most important treatment for anaphylaxis . It has alpha - 1 and beta - 2 agonist effects that quickly cause vasoconstriction , increasing blood pressure , and bronchodilation , improving wheezing . Epinephrine also decreases further systemic release of inflammatory mediators , halting anaphylaxis . Intramuscular injection , performed rapidly , is the preferred route of administration and can be repeated every 5-15 - minutes ( up to 3 doses ) if symptoms persist . ( Choice A ) Continuous inhaled albuterol may relieve bronchospasm in this patient . Unlike epinephrine , it would not improve hypotension and prevent impending circulatory collapse . ( Choice B ) Endotracheal intubation is performed when prominent upper airway edema ( eg , oropharyngeal swelling , stridor ) is present in anaphylaxis ; it does not directly address lower airway bronchospasm , the source of this patient's respiratory distress . In contrast , rapid epinephrine administration may improve bronchospasm and prevent progression to respiratory failure . ( Choice D ) Intravenous glucocorticoids ( eg , methylprednisolone ) are frequently administered as adjunct treatment for anaphylaxis due to their anti - inflammatory effect . However , onset is delayed by several hours , and they do not provide immediate lifesaving treatment ( as epinephrine does ) for hypotension and respiratory distress . ( Choice E ) Intravenous fluids are used as an adjunct to epinephrine to treat hyp

A 20 - year - old woman comes to the office due to nausea . She has had a decreased appetite and felt " queasy " for the past 2 weeks . She has had no vomiting , abdominal pain , or diarrhea . The patient's last menstrual period was 9 weeks ago . Vital signs and physical examination are normal . A pregnancy test is positive and urinalysis is negative for ketones . Ultrasound confirms a viable intrauterine gestation . Which of the following is the best next step in management of this patient's nausea ? A. Dietary modification B. Doxylamine C. Hydroxyzine D. Ondansetron E. Prochlorperazine

A. Dietary modification This patient has mild nausea of pregnancy , which affects most pregnant women during the first trimester and usually subsides by 16-20 weeks gestation . In pregnancy , nausea can occur with or without vomiting and can be mild , moderate , or severe . Risk factors include multifetal gestation , gastroesophageal reflux , and history of migraines . Treatment is based on symptom intensity and frequency , and how the patient's quality of life is affected . Dietary modification is first - line treatment for mild nausea of pregnancy . Patients should avoid offending smells or foods and eat small , frequent meals ( eg , bland carbohydrates , protein - based snacks ) ; ginger may also be helpful . Patients who do not improve with these measures are started on vitamin B , ( ie , pyridoxine ) with or without doxylamine succinate ( Choice B ) . Hospitalization for intravenous fluids is reserved for patients with dehydration ( eg , ketonuria , hypokalemia ) secondary to hyperemesis gravidarum . ( Choice C ) Hydroxyzine , a potent first - generation antihistamine , is contraindicated in pregnancy as it crosses the placenta and may cause neonatal withdrawal . ( Choice D ) Safety data regarding ondansetron in pregnancy is conflicting ; some studies suggest that it may increase the risk of fetal cardiac septal defects . Therefore , it is a third - line treatment option . Choice E ) Prochlorperazine , a phenothiazine dopamine antagonist , is used for severe nausea and vomiting ( eg , hyperemesis gravidarum ) . However , it is considered third - line therapy due to risks of QT prolongation , sedation , and limited fetal safety data . Educational objective : Dietary modification is first - line treatment for mild nausea of pregnancy . Pregnant women should avoid offending odors / foods and eat small , frequ

Acute diverticulitis Diagnosis .

Abdominal CT scan ( oral & intravenous contrast ) This patient with abdominal pain , leukocytosis , low - grade fever , and mass in the left lower quadrant likely has acute diverticulitis . Diverticulitis is characterized by inflammation of colonic diverticula ( ie , herniated outpouchings of the colonic mucosa and submucosa through the muscularis mucosae ) and is caused by trapped food particles and elevated intraluminal pressure within a diverticulum , resulting in microperforation . The risk increases with age , particularly in those age 260 . Patients commonly have constant abdominal pain , classically in the left lower quadrant , signifying sigmoid colon involvement . Other symptoms include nausea , vomiting , and changes in bowel habits ( constipation or diarrhea ) . Patients often have low - grade fever , and examination may reveal lower abdominal tenderness and , often , a palpable mass due to inflammation or abscess formation . Mild leukocytosis is common ; occult blood may be present , but gross hematochezia is rare . Diagnosis is confirmed with CT scan of the abdomen , and treatment is based on severity ( eg , outpatient oral vs inpatient intravenous antibiotics ) .

A 25 - year - old woman is evaluated for a worsening rash . The patient returned from a camping trip 2 days ago and developed an itchy , erythematous rash on her upper extremities and torso . Some of the lesions progressed into small blisters . She reports falling into a bush during the trip . Vital signs are normal . Skin examination findings are shown in the exhibit . The rest of the examination is normal . Which of the following is the most likely diagnosis ? Acute allergic contact dermatitis Dermatitis herpetiformis Rocky Mountain spotted fever Staphylococcal toxic shock syndrome Stevens - Johnson syndrome

Acute allergic contact dermatitis

Ovarian torsion Ultrasonography findings

Adnexal mass with absent Doppler flow to ovary

A 22 - year - old woman presents to the emergency department after spilling hot tea on her left forearm . She reports pain and redness in the area . Physical exam shows a red , painful , and tender area without blister formation . What is the most likely diagnosis ? A Deep partial - thickness burn B Full - thickness burn C Superficial burn D Superficial partial - thickness burn

Answer C. A superficial burn ( also known as first degree burn ) is characterized by erythema , pain , and tenderness without blister formation . The area typically looks similar to a sunburn . These types of burns usually heal within seven days and do not leave any areas . of scarring . Treatment for this degree of burn is symptomatic . Nonsteroidal anti - inflammatory drugs can be used to decrease pain . Topical analgesics can also be used for localized anesthesia . Second degree burns are better characterized as superficial or deep partial thickness burns . In superficial partial - thickness burns , the superficial dermis is also injured . There is blistering of the skin and pain with palpation . These burns usually heal within 14 to 21 days and there may or may not be scarring . Deep partial - thickness burns extend into the reticular layer of the dermis . There will be damage to the sweat and sebaceous glands . The skin will demonstrate blistering and the exposed dermis will be pale in color . Healing takes up to two months and scarring is likely . Full - thickness burns ( also known as third degree burns ) , destroy all epidermal and dermal structures . These are typically painless . Healing will not take place spontaneously and skin grafts and surgical repair are necessary Treatment is dependent on the level of burn injury and location of burn . Fluid resuscitation , wound care , pain control , and management of complications is the mainstay of therapy .

A 29 - year - old man is evaluated for right eye pain , redness , and blurry vision for the past 2 days . Physical examination findings are shown . No discharge is present . Which of the following is the most likely diagnosis ? Acute - angle - closure glaucoma Anterior uveitis Bacterial conjunctivitis Episeleritis Herdeelum

Anterior uveitis

Pemphigus vulgaris Laboratory evaluation

Antidesmosomal ( desmoglein ) antibodies • Biopsy : IgG throughout epidermis

A 78 - year - old man is found unresponsive and is brought by ambulance to the emergency department . His family says that he has a complicated medical history and takes multiple medications but has no known drug allergies . Noncontrast CT scan of the head reveals an intracranial hemorrhage . The patient is given a modified , inactive form of factor Xa . This agent is most likely to antagonize the effects of which of the following medications ? A. Apixaban B. Aspirin C. Dabigatran D. Ticagrelor E. Warfarin

Apixaban

A 35 - year - old woman is brought to the emergency department after being rescued from inside a burning building . The patient had a brief tonic - clonic seizure ; she s confused and mildly agitated . Temperature is 37 C ( 98.6 F ) , blood pressure is 100/60 mm Hg , pulse is 115 / min , and respirations are 24 / min . Oxygen saturation is 96 % on room air as measured by standard pulse oximetry . Physical examination shows no burns , and her skin color is normal . There are symmetric breath sounds bilaterally with scattered end expiratory wheezes . Neurologic examination shows no abnormalities apart from confusion . The abdomen is soft and nontender . Which of the following is the best initial treatment for this patient ? A . 50 % dextrose with thiamine B. 100 % oxygen with facemask C. Intravenous lorazepam D. Intravenous nalexene E. Intravenous phenytoin

B. 100 % oxygen with facemask This patient presents with confusion , wheezing , and a seizure following smoke inhalation ; this clinical picture suggests carbon monoxide ( CO ) poisoning . CO is a tasteless , colorless , and odorless gas produced by incomplete combustion of carbon - containing compounds . CO poisoning should be considered in all patients exposed to smoke in a closed space . The affinity of CO for binding hemoglobin ( Hb ) is > 200 times that of oxygen ; once bound to Hb , CO forms carboxyhemoglobin , which impairs oxygen delivery to tissue by shifting the Hb - oxygen dissociation curve to the left . Manifestations of mild to moderate CO toxicity include headache , nausea , dyspnea , malaise , altered mentation , and dizziness . Severe CO poisoning can present with seizures , coma , syncope , heart failure , or arrhythmias . Bright cherry lips can be a sign of CO poisoning ( not specific ) . The diagnosis is confirmed clinically and by documenting an elevated carboxyhemoglobin level ( eg , > 3 % in nonsmokers , > 10 % in smokers ) . A standard pulse oximetry is unreliable and may appear normal because it cannot differentiate carboxyhemoglobin from oxyhemoglobin ( as seen in this patient ) . The treatment of CO poisoning involves administration of 100 % oxygen via nonrebreather facemask to compete with CO binding to Hb and to decrease the half - life of CO ( from -5 hours on room air to 1-2 hours on 100 % oxygen ) . Patients should then be monitored ( for 24 hours ) and hospitalized if their condition has not improved . Hyperbaric oxygen is sometimes used in severe cases that are unresponsive to facemask - administered oxygen . ( Choice A ) A bolus of 50 % dextrose can be given for suspected hypoglycemia in an unconscious patient . Intravenous thiamine can be given for suspected Wernicke encephalopath

A 66 - year - old man comes to the emergency department due to a week of progressive worsening exertional dyspnea accompanied by a dry cough . Medical history includes hypertension , recent stenting for double - vessel coronary artery disease , congestive heart failure ( CHF ) , and chronic obstructive pulmonary disease ( COPD ) . He was hospitalized 6 months ago for pneumonia . He has a 35 - pack year smoking history . Temperature is 37.2 C ( 98.9 F ) , blood pressure is 160/90 mm Hg , and pulse is 90 / min and regular . On examination , the patient is in mild respiratory distress , but he can speak in full sentences . Chest auscultation reveals decreased breath sounds at the lung bases , bilateral crackles , and occasional wheezes . Arterial blood gas shows : pH 7.46 PO₂ 73 mm Hg pCO₂ 31 mm Hg Which of the following is the most likely explanation for this patient's symptoms ? A. Acute respiratory distress

B. CHF exacerbation This patient with shortness of breath has risk factors for both congestive heart failure ( CHF ) and chronic obstructive pulmonary disease ( COPD ) exacerbations ; however , several elements of his clinical presentation suggest he most likely has a CHF exacerbation . His history of coronary artery disease , with risk factors of uncontrolled hypertension and smoking , can lead to CHF due to left ventricular dysfunction . Patients with worsening CHF commonly have bibasilar crackles . Decreased breath sounds at the bases could be due to bilateral pleural effusions . Wheezing can occasionally be present ( ie , cardiac asthma ) . Arterial blood gas ( ABG ) can show hypoxia ( due to fluid pooling in the lungs ) as well as hypocapnia and respiratory alkalosis ( due to tachypnea ) . Other manifestations of CHF ( not noted in this patient ) include S3 and S4 gallops and cardiomegaly . Although his extensive smoking history may have caused COPD , a COPD exacerbation is less likely to cause bibasilar crackles , more likely to cause significant rather than occasional wheezing , and more likely to cause hypercapnia and respiratory acidosis ( due to CO₂ retention ) on ABG ( Choice C ) . The hypercapnia is thought to be due to airway obstruction ( air trapping ) and loss of elastic recoil from the damaged airways . More specific testing for CHF would include assessment of the B - type natriuretic peptide ( BNP ) , which would generally be elevated in CHF but not in COPD . Choice A ) Acute respiratory distress syndrome can have a similar presentation but tends to be acute . This patient's history of progressively worsening dyspnea and his risk factors of coronary artery disease suggest CHF as the most likely diagnosis . Choice D ) Pneumothorax typically presents with acute - onset pleuritic chest pain and dyspn

A 35 - year - old , previously healthy man is brought to the emergency department after he fell off the roof of his house . The patient has significant blunt chest and head trauma . Shortly after arrival , blood pressure drops suddenly , and he experiences respiratory distress . On physical examination , the patient is tachycardic and tachypneic . The lungs are clear to auscultation with vesicular breath sounds in all lung fields . He has jugular venous distension , and systolic blood pressure falls 15 mm Hg with inspiration . Which of the following is the most likely cause of this patient's deterioration ? A. Aortic rupture B. Cardiac tamponade C. Epidural hematoma D. Hemothorax E. Tension pneumothorax

B. Cardiac tamponade

A 34 - year - old man comes in due to chest pain for the past 3 months . The patient describes frequent episodes of substernal , squeezing - type chest discomfort that radiates to his neck . The episodes are not related to any particular activity , often occur at rest , and consist of constant pain lasting up to 2 hours . He has no dyspnea , wheezing , palpitations , syncope , or leg swelling . The patient does not use tobacco , alcohol , or recreational drugs . He works as an airline pilot . Family history is unremarkable . Blood pressure is 118/72 mm Hg , and pulse is 76 / min . BMI is 30 kg / m2 . ECG is normal . Exercise stress test shows no abnormalities . Which of the following is the most likely cause of this patient's chest pain ? XOA . Aortic disease B. Goronary artery disease C. Esophageal disease D. Musculoskeletal disease E. Pleural disease

C. Esophageal disease This young patient with recurrent , prolonged chest pain at rest and a negative cardiac evaluation ( eg , stress test , ECG ) likely has esophageal chest pain . Esophageal chest pain ( eg , from gastroesophageal reflux disease ) is the most common mimicker of angina and is the second most common cause of chest pain in the primary care population ( after musculoskeletal pain ) . Often , esophageal chest pain can be distinguished from angina based on the history : • Timing : Esophageal chest pain lasts longer ( eg , hours ) ; angina typically lasts minutes . • Precipitating factors : Esophageal chest pain is precipitated by meals or the recumbent position ( eg , resting ) ; angina is typically precipitated by exertion . • Associated features : Other features , including regurgitation , dysphagia , and improvement with antacids , are often present in esophageal chest pain but not with angina . In contrast , pain radiation is a less reliable distinguishing factor . Pain fibers from the esophagus and heart ( visceral afferents ) overlap at multiple different spinal cord levels , where they converge with somatic nerves . Therefore , chest pain caused by esophageal or cardiac pathology can be referred to the arms , jaw , neck , and / or back . ( Choice A ) Aortic dissection typically presents with sudden - onset , severe , persistent chest or back pain . Given this patient's chronic , episodic symptoms and absence of risk factors ( eg , hypertension ) , aortic dissection is unlikely . ( Choice B ) Atherosclerotic and vasospastic ( eg , Prinzmetal angina ) coronary artery diseases typically cause chest pain that lasts for minutes ; chest pain lasting for hours suggests a noncardiac source . ( Choice D ) Musculoskeletal pain is typically sharp , is localized to a specific area over the chest wa

A 43 - year - old man is brought to the emergency department after suffering serious injuries during a fight at a local bar . The patient smells heavily of alcohol , slurs his words , and has difficulty standing and walking . It is unclear if he experienced a loss of consciousness . Physical examination reveals numerous contusions and lacerations on his face that will require suturing . The patient has unilateral pupil dilation in the right eye and appears sleepy . When he is informed that he will need a head CT scan , he rouses and becomes belligerent . The patient begins to shout and curse , claiming that the staff is harassing him and that he will sue the hospital for violation of his civil rights . Which of the following is the most appropriate course of action ? A. Contact the patient's next of kin to obtain consent B. Discharge the patient if he signs a release of liability C. Immediately perform the head C

C. Immediately perform the head CT scan

A 23 - year - old woman is brought to the emergency department due to an hour of difficulty breathing . She has a history of asthma and has been hospitalized several times with exacerbations . The patient is not compliant with an inhaled corticosteroid but takes albuterol as needed . She says that the breathing difficulty feels like an asthma attack . The patient was diagnosed with acute sinusitis for which she took an antibiotic earlier this morning . She has used the albuterol inhaler several times since symptom onset . Blood pressure is 82/50 mm Hg , pulse is 118 / min , and respirations are 30 / min . Oxygen saturation is 85 % on room air . Chest auscultation reveals audible wheezing bilaterally and prolonged expiration . An urticarial rash is seen over the trunk and upper extremities , and the extremities are warm . Which of the following is the best next step in management of this patient ? A. Continuous inha

C. Intramuscular epinephrine This patient with skin findings ( eg , urticarial rash ) , respiratory difficulty ( eg , wheezing ) , and hypotension has anaphylaxis , likely caused by antibiotic exposure . Diagnosing anaphylaxis can be challenging if symptoms are initially attributed to a preexisting medical condition ( eg , asthma ) , as with this patient . Epinephrine is the most important treatment for anaphylaxis . It has alpha - 1 and beta - 2 agonist effects that quickly cause vasoconstriction , increasing blood pressure , and bronchodilation , improving wheezing . Epinephrine also decreases further systemic release of inflammatory mediators , halting anaphylaxis . Intramuscular injection , performed rapidly , is the preferred route of administration and can be repeated every 5-15 - minutes ( up to 3 doses ) if symptoms persist . ( Choice A ) Continuous inhaled albuterol may relieve bronchospasm in this patient . Unlike epinephrine , it would not improve hypotension and prevent impending circulatory collapse . ( Choice B ) Endotracheal intubation is performed when prominent upper airway edema ( eg , oropharyngeal swelling , stridor ) is present in anaphylaxis ; it does not directly address lower airway bronchospasm , the source of this patient's respiratory distress . In contrast , rapid epinephrine administration may improve bronchospasm and prevent progression to respiratory failure . ( Choice D ) Intravenous glucocorticoids ( eg , methylprednisolone ) are frequently administered as adjunct treatment for anaphylaxis due to their anti - inflammatory effect . However , onset is delayed by several hours , and they do not provide immediate lifesaving treatment ( as epinephrine does ) for hypotension and respiratory distress . ( Choice E ) Intravenous fluids are used as an adjunct to epinephrine to treat hypotension

A 68 - year - old man with type 2 diabetes mellitus is brought to the emergency department due to slurred speech and weakness for the last 12 hours . Three days ago , the patient completed a course of oral prednisone due to severe poison ivy . Temperature is 36.7 C ( 98.1 F ) , blood pressure is 99/80 mm Hg , and pulse is 99 / min . On examination , the patient is oriented only to place and is unable to consistently follow commands . Mucous membranes are dry . Neurologic examination reveals dysarthria and muffled speech . Laboratory results are as follows : Sodium 128 mEq / L Potassium 4.8 mEq / L Chloride 98 mEq / L Bicarbonate 24 mEq / L Blood urea nitrogen 39 mEq / L Glucose 911 mg / dL CT scan of the head is normal . Intravenous access is secured , and an infusion of normal salin and regular insulin is started . Which of the following is the best next step in management of this patient ? A. Administration of ti

C. Intravenous potassium

A 59 - year - old man comes to the office due to diplopia and a drooping right eyelid . His symptoms began a day earlier and have since worsened . Physical examination shows a dilated right pupil that is nonreactive to both light and accommodation . There is vertical and horizontal diplopia . When the patient is asked to stare straight ahead , the right eye is directed inferiorly and laterally with respect to the left eye . MRI of the brain reveals an aneurysm involving the right posterior communicating artery . Which of the following muscles is most likely to remain functionally intact in this patient ? A. Inferior oblique B. Inferior rectus C. Lateral rectus D. Levator palpebrae E. Medial rectus

C. Lateral rectus This patient has a right oculomotor nerve ( CN III ) palsy secondary to a compressive aneurysm . Lesions involving CN III cause ptosis ( drooping of the upper eyelid ) and impair adduction , depression , and elevation of the eye . As a result , diagonal ( eg , horizontal and vertical ) diplopia is frequently present . The eye rests in a " down and out " position due to the unopposed actions of the superior oblique and lateral rectus muscles , which are innervated by the trochlear nerve ( CN IV ) and the abducens nerve ( CN VI ) , respectively , and are therefore unaffected by CN III palsy . Pupillary constriction and accommodation can also be affected as CN III carries parasympathetic fibers to the ciliary muscle and the iris sphincter . CN III palsy can be due to a variety of causes , including intracranial tumors or aneurysms , diabetic mononeuropathy , and head trauma . Aneurysms causing CN III palsy most often involve the posterior communicating artery . When due to an aneurysm , symptoms that develop acutely are especially worrisome as they suggest active aneurysmal dilation and possible impending rupture . ( Choices A , B , D , and E ) All of these muscles are innervated by CN III , so their function will be impaired in this patient . Educational objective : Lesions involving the oculomotor nerve ( CN III ) cause ptosis , a downward and laterally deviated eye , impaired pupillary constriction and accommodation , and diagonal diplopia . The most dreaded cause of CN III palsy is an enlarging intracranial aneurysm .

A 16 - year - old boy is seen due to right knee pain after a fall . Symptoms started after a skiing accident in which he caught the inside edge of his ski and fell . The patient has no medical conditions and takes no medications . On examination , pain and joint laxity is elicited when valgus stress is applied to the knee with the lower leg kept stationary . Which of the following structures is most likely injured in this patient ? A. Anterior cruciate ligament B. Lateral collateral ligament C. Medial collateral ligament D. Medial meniscus E. Posterior cruciate ligament

C. Medial collateral ligament This patient has an injury to the medial collateral ligament ( MCL ) . The MCL attaches proximally to the medial epicondyle of the femur and inserts on the medial proximal tibia . It resists forces that push the knee medially . MCL injury most commonly occurs from valgus stress or a direct blow or to the lateral knee while the foot is planted . It is a common ligamentous knee injury in skiing and in sports that require sudden changes of direction ( eg , football , tennis ) . Physical examination may show swelling or ecchymosis . Tenderness at a specific point along the length of the ligament or at an insertion point often indicates the site of injury . The valgus stress test is the primary method for testing MCL integrity ; it is performed by placing one hand along the lateral knee and pressing inward while the other hand holds the medial aspect of the ankle and pulls outward . Laxity of the knee or medial joint line opening indicates MCL injury . ( Choice A ) The anterior cruciate ligament resists anterior movement and medial rotation of the tibia . Injury is indicated by increased anterior movement of the tibia in relation to the femur ( eg , anterior drawer test ) , Lachman test ) . ( Choice B ) The lateral collateral ligament ( LCL ) resists excessive lateral movement of the knee . LCL injury is uncommon but would be characterized by laxity of the knee with varus stress testing . ( Choice D ) The medial and lateral menisci reduce shock and help transfer force from the upper to the lower leg . Medial meniscal injury can occur with twisting movement of the knee and often accompanies MCL injury . Lateral meniscal injury is less common . Meniscal injury can cause joint line tenderness but not laxity ( or opening of the joint line ) with valgus stress testing . In addition , affecte

A 22 - year - old woman comes to the emergency department due to acute vision disturbances and headache . About an hour ago the patient experienced a small area of visual loss , which she describes as a hole in the visual field . It subsequently gradually expanded to a C - shape in the peripheral vision and spontaneously resolved after 15-20 minutes . A progressively worsening , severe , right - sided headache of pulsating quality quickly followed . The patient has had similar headaches in the past but not of such severity . Blood pressure is 150/86 mm Hg and pulse is 92 / min . On physical examination , the patient is lying still in bed with eyes closed . The pupils are equal and reactive to light . Funduscopic examination reveals no abnormalities . Bilateral upper and lower extremity muscle strength , deep tendon reflexes , and sensation are normal . Which of the following is the most likely diagnosis ? A. Aeute

C. Migraine with aura

A 6 - year - old girl is evaluated for a prolonged cough . The patient started having rhinorrhea 6 weeks ago . A week later , she developed a cough with occasional expectoration of whitish sputum ; the rhinorrhea resolved at that time . For the past 5 weeks , the patient's cough has continued throughout the day and night , occasionally awakening her and showing no improvement . She has been afebrile with no sick contacts . Immunizations are up to date . The patient's mother has atopic dermatitis . Height and weight are at the 35th percentile . Temperature is 36.7 C ( 98.1 F ) and respirations are 24 / min . Physical examination is within normal limits . Which of the following is the best next step in evaluation of this patient ? A. CT scan of the chest ( 1 % ) B. PCR for Bordetella species ( 13 % ) C. Spirometry ( 67 % ) D. Sputum culture ( 5 % ) E. Sweat chloride level ( 12 % )

C. Spirometry Chronic cough in children is defined as cough lasting > 4 weeks . Initial evaluation includes a detailed history and thorough physical examination to potentially identify a specific underlying cause . For example , poor growth may suggest a chronic illness ( eg , cystic fibrosis [ CF ] ) , or peripheral edema may suggest cardiovascular disease . In the absence of specific historical ( eg , concomitant upper respiratory symptoms ) or examination findings , initial evaluation begins with spirometry . Spirometry is a simple pulmonary function test that can be performed in a cooperative child to detect obstructive airway disease ( eg , asthma ) . Cough due to asthma may be associated with wheezing , shortness of breath , or a history of atopy , as in this child with a family history of atopic dermatitis . However , isolated chronic cough with a normal lung examination may be the sole finding in asthma . Chest x - ray is also recommended for the evaluation of chronic cough . It may be helpful in identifying abnormalities such as mediastinal widening ( eg , chronic infection , neoplasm ) , hyperinflation ( eg , foreign body aspiration ) , or cardiac defects . Choice A ) CT scan of the chest in children is typically reserved for evaluation of abnormal findings ( eg , mass ) found on less invasive imaging ( eg , x - ray ) due to radiation exposure . This child warrants imaging if spirometry is normal , but x - ray is indicated first . Choice B ) Pertussis can present with a chronic cough following the paroxysmal stage and classically presents with a " whoop " in children age < 10 . Vaccinated children are less likely to develop pertussis and typically have a less severe illness . Pertussis testing could be considered if spirometry and imaging are normal . ( Choice D ) Sputum culture is obtained for patien

A 68 - year - old man comes to the emergency department due to chest pain . The patient was hospitalized 2 weeks ago for a non - ST elevation myocardial infarction which required placement of a drug - eluting stent . The patient has been taking his medications and has limited his physical activity to prevent overexertion . Last night he began having dyspnea and sharp , left - sided chest pain , which worsens with deep inspiration . Temperature is 38.1 C ( 100.6 F ) , blood pressure is 116/84 mm Hg , pulse is 110 / min , and respirations are 20 / min . Oxygen saturation is 92 % on room air . The patient appears in mild respiratory distress . The lungs are clear on auscultation and there are no cardiac murmurs . There is mild right lower extremity edema . The distal pulses are full . ECG shows sinus tachycardia with nonspecific T wave changes . Troponin I is 0.4 ng / mL ( normal : < 0.01 ) . Chest x - ray is normal . W

CT pulmonary angiography Pulmonary embolism ( PE ) is a relatively common , life - threatening cause of pleuritic chest pain and should be strongly considered in patients with risk factors ( eg , recent hospitalization with prolonged immobilization ) . The evaluation of suspected PE requires an assessment of pretest probability . This patient has several features that are highly suggestive of PE including dyspnea , tachycardia , tachypnea , and unilateral lower extremity edema . Pleural effusion , low - grade fever ( due to inflammation from tissue damage ) , and troponin elevations ( due to myocardial ischemia ) can also be seen . Using the modified Wells criteria , this patient has a high ( likely ) probability for PE ( heart rate > 100 / min , recent hospitalization / prolonged immobilization , alternate diagnosis less likely than PE ) , and CT pulmonary angiography is indicated as it has high diagnostic accuracy in such patients . D - dimer testing is useful in excluding PE in patients with low pretest probability , but it has no utility in patients with high pretest probability due to an unacceptable rate of false - negative results in this population . Choices B , D , and E ) This patient is at risk for recurrent acute coronary syndrome ( ACS ) . Emergency coronary angiography is indicated for ST - elevation myocardial infarction ( MI ) ( eg , due to stent thrombosis ) ; however , this should present with ST elevation on ECG and ischemic ( ie , pressure - like , unchanged with breathing ) chest pain , rather than pleuritic chest pain . Serial ECGS and troponin levels or stress testing are indicated for suspected non - ST elevation MI or unstable angina , but these ACS manifestations should also present with ischemic , rather than pleuritic , chest pain . ( Choice C ) Postcardiac injury ( Dressler ) syndrome is

A 29 - year - old woman , gravida 1 para 1 , is evaluated in the emergency department for a headache . Three days ago , she had an uncomplicated vaginal delivery with neuraxial anesthesia . This morning , the patient developed an occipital headache that has not improved with acetaminophen . The headache has become progressively worse , and the patient cannot sit without becoming nauseated and vomiting . Temperature is 36.7 C ( 98.1 F ) , blood pressure is 162/96 mm Hg , and pulse is 98 / min . On examination , there is 5/5 strength in the right lower extremity and 2/5 strength in the left . Deep tendon reflexes are 3+ . Which of the following is the best next step in management of this patient ? 100 % oxygen ( 4 % ) CT scan of the head ( 54 % ) Epidural blood patch ( 29 % Lumbar puncture ( 9 % ) Sumatriptan therapy ( 2 % )

CT scan of the head ( 54 % ) This postpartum patient with worsening headache and severe hypertension ( eg , systolic blood pressure 2160 or diastolic blood pressure 2110 mm Hg ) has preeclampsia with severe features , which can present up to 6 weeks after delivery . The most common presenting symptom is a severe headache in the bilateral occipital or frontal regions that does not improve with acetaminophen or nonsteroidal anti - inflammatory drugs . Patients with preeclampsia are at increased risk of hemorrhagic and ischemic stroke due to acute elevations in cerebral perfusion pressure and vessel rupture ( hemorrhagic ) , as well as preeclampsia - mediated vascular endothelial damage and microthrombi formation ( ischemic ) . To decrease this risk , preeclamptic patients with severe - range blood pressures require aggressive antihypertensive therapy ( eg , labetalol , nifedipine ) and magnesium sulfate , which helps prevent eclamptic seizures that can worsen stroke symptoms . Most preeclamptic patients do not require imaging ; however , in those with focal neurologic deficits , such as this patient with asymmetric motor deficits ( strength right > left ) , CT scan of the head should be performed to evaluate for possible stroke and help guide management . Choices A and E ) Oxygen administration is used to alleviate cluster headaches , which typically present with a short - lived , unilateral , orbital headache with autonomic symptoms ( eg , ptosis , lacrimation ) . Triptans ( eg , sumatriptan ) are used to treat migraines , which can present as occipital headaches with nausea , vomiting , and occasionally motor aura ( eg , hemiplegic migraine ) . However , CT scan should be performed in this patient prior to attempting symptomatic therapy to exclude life - threatening causes of neurologic deficit . ( Choice C ) An epidu

A 25 - year - old man is brought to the emergency department after he fell headfirst down a flight of stairs while intoxicated . According to friends who witnessed the fall , the patient landed on his head but did not lose consciousness . On arrival , blood pressure is 128/78 mm Hg , pulse is 92 / min , and respirations are 12 / min . The patient smells of alcohol and is somnolent but can follow simple commands . A rigid cervical collar is in place . The patient moves all extremities spontaneously . He states that he is uncomfortable due to the cervical collar and requests that it be removed . Which of the following is the best next step for evaluating and ruling out cervical spine injury in this patient ? CT scan without contrast Flexion and extension x - rays MRI without contrast Portable lateral x - ray Thorough neurologic examination

CT scan without contrast

A 45 - year - old man comes to the emergency department due to a 1 - day history of constant upper abdominal pain , nausea , and several episodes of vomiting . The pain is partially relieved by sitting up or bending forward . He has also had mild dyspnea . The patient's other medical conditions include recently diagnosed gallstones . Temperature is 38.1 C ( 100.6 F ) , blood pressure is 110/70 mm Hg , pulse is 114 / min , and respirations are 22 / min . Abdominal examination shows epigastric tenderness . Laboratory results are as follows : Hematocrit 44 % Leukocytes 16,000 / mm³ Calcium 7.8 mg / dL Glucose 250 mg / dL Which of the following is the best next step in management of this patient's current condition ? A. Antiplatelet agents B. Broad - spectrum antibiotics C. Insulin infusion D. Isotonic crystalloids E. Proton pump inhibitors

D. Isotonic crystalloids This patient's presentation - nausea , vomiting , and constant epigastric abdominal pain partially relieved by leaning forward - indicates acute pancreatitis . Common risk factors for acute pancreatitis include gallstones ( as in this patient ) , alcohol use , and hypertriglyceridemia . Activated pancreatic enzymes ( eg , phospholipase , trypsin ) and cytokines ( eg , tumor necrosis factor ) released from the inflamed pancreas into the circulation can lead to systemic inflammation . As a result , similar to sepsis , the best next step in management is aggressive volume resuscitation with isotonic crystalloids . Patients have significantly lower morbidity and mortality rates when appropriate volume replacement occurs in the first 48 hours as it helps to minimize hypotension , end - organ damage , and necrotizing pancreatitis . In addition to aggressive volume resuscitation , the initial management of acute pancreatitis involves pain management and close monitoring for complications ( ie , infection , renal failure , acute respiratory distress syndrome ) . ( Choice A ) Antiplatelet agents are appropriate in a patient having a myocardial infarction . These patients can sometimes present atypically with nausea , vomiting , and epigastric pain ( especially an inferior myocardial infarction ) . However , fever and pain that improves while sitting up or leaning forward are not typical . ( Choice B ) Patients with acute pancreatitis can be febrile and have a leukocytosis , so an infectious workup ( eg , blood cultures , chest x - ray ) should be considered ; however , broad - spectrum antibiotics are generally not given empirically unless there is evidence of infected pancreatic necrosis ( eg , on CT scan ) or an extrapancreatic infection ( eg , pneumonia ) . ( Choice C ) Patients with acute pancreat

A 45 - year - old man comes to the urgent care after sudden onset of a skin rash and oral lesions with fever . The patient is unable to eat or drink due to pain in his mouth and throat . He had a headache and malaise prior to developing the skin lesions . The patient had an episode of sinusitis for which he was prescribed trimethoprim - sulfamethoxazole 5 days ago . Temperature is 38.3 C ( 101 F ) , blood pressure is 110/80 mm Hg , pulse is 92 / min , and respirations are 14 / min . On examination , conjunctivae in both eyes are inflamed . The patient has extensive erosions involving the oral mucosa . There is also an erythematous , desquamating rash with scattered involvement of the trunk and proximal thighs , as shown above . Which of the following is the most likely diagnosis ? A. Erythema multiforme B. Impetigo C. Pemphigus vulgaris OD . Stevens - Johnson syndrome E. Toxic shock syndrome

D . Stevens - Johnson syndrome

A 21 - year old man is brought to the emergency department due to head trauma sustained during a boxing match . During the fight , he had a brief loss of consciousness after a hard hit to the right side of his face . He has a headache and muffled hearing in his right ear . On examination , the patient is alert and oriented . Palpation of the head does not reveal any step - offs . Cervical spine examination is normal . Otoscopic examination reveals hemotympanum in the right ear . This patient is at greatest risk for injury involving which of the following cranial nerves ? A. Olfactory nerve B. Oculomotor nerve C. Trigeminal nerve D. Facial nerve E . Hypoglossal nerve

D. Facial nerve This patient who sustained trauma to the head now has hearing loss and hemotympanum , which is concerning for a temporal bone fracture . Contained within the temporal bone are many structures involved in hearing , including the external auditory canal , tympanic membrane , ossicular chain , cochlea , and vestibulocochlear nerve ( CN VIII ) . Injury or disruption to any of these structures can cause hearing loss , as in this patient . The temporal bone also contains the facial nerve ( CN VII ) , which originates in the brainstem and then enters the temporal bone at the internal auditory meatus . It travels through the internal auditory canal with the vestibulocochlear nerve ( CN VIII ) and then takes a complicated route through the temporal bone itself , traveling near the otic capsule , through the middle ear , and then through the mastoid portion to exit at the stylomastoid foramen . Because of its location , the facial nerve is at risk of injury in temporal bone fractures . Deficits can be diverse and significant because the facial nerve has many functions , including : • Movement of the face , including eye closure • Taste from the anterior 2/3 of the tongue . • Parasympathetic innervation of submandibular and lacrimal glands • Sound dampening ( due to innervation of the stapedius muscle ) ( Choice A ) Axons of chemoreceptor cells make up the olfactory nerve ( CN I ) rootlets , which project through the cribriform plate of the ethmoid bone . Acceleration - deceleration head injuries can lead to avulsion of the olfactory nerve rootlets , but this would not be associated with hearing loss or hemotympanum . ( Choice B ) The oculomotor nerve ( CN III ) exits the skull base from the superior orbital fissure and is at risk with trauma to the orbit or anterior face . ( Choice C ) The trigeminal ne

A 20YO previously healthy woman is brought to the ER due to lethargy. According to her mother, the pt had an episode of nausea, vomiting, and abd pain that began 2 days ago after eating in a restaurant. Her vomiting has resolved, but she continues to have nausea and abd pain. ROS is positive for excessive thirst and a 9lb weight loss over the past 2 weeks. PT does not use tobacco, alcohol or illicit drugs. T is 99, BP is 100/56, P is 120, and RR are 28. PT is lethargic but easily arousable; responds to questions in short sentences and moves all extremities spontaneously. Mucous membranes are dry. There is diffuse abd tenderness without rebound tenderness, guarding, or rigidity. Which of the following is the best next test? A. CBC B. ABD US C. ABG D. Fingerstick glucose

D. Fingerstick glucose This patient's presentation suggests diabetic ketoacidosis ( DKA ) from new - onset type 1 diabetes mellitus ( T1DM ) . DKA was likely triggered by her recent gastrointestinal illness ( ie , nausea and vomiting ) , which exacerbated dehydration and precipitated ketosis . Features of this patient's illness consistent with DKA include : • Young age : Although T1DM is classically associated with childhood onset , one - fourth of new cases are diagnosed in young adults . DKA is the second most common initial presentation of T1DM . Symptoms : Rapid onset of weight loss , polydipsia ( increased thirst ) , and diffuse abdominal pain occurs , possibly from acidosis and electrolyte abnormalities impairing gastrointestinal function ( eg , delayed gastric emptying , ileus ) . • Clinical signs : Dehydration ( evidenced by tachycardia and hypotension ) and hyperventilation ( ie , compensation for metabolic acidosis ) are prominent . Therefore , checking fingerstick glucose is the best next step in management . This test is rapid and minimally invasive ; it can . narrow the differential diagnosis and initial treatment for this patient's gastrointestinal symptoms and lethargy . Glucose levels of 300 800 mg / dL should prompt empiric treatment for DKA ( eg , insulin , intravenous fluids ) and confirmatory testing ( eg , serum ketones , arterial blood gas ) . ( Choice A ) Although infection is a common trigger of DKA , blood glucose measurement should be performed first because it can rapidly confirm a presumptive diagnosis and guide management . Choice B ) Abdominal pain , hypotension , and vomiting can be seen in primary adrenal insufficiency , diagnosed via cosyntropin stimulation testing . However , adrenal insufficiency is unlikely to explain polydypsia and hyperventilation .

A 22 - year - old woman comes to the emergency department due to acute - onset nausea , diarrhea , and shortness of breath . She says , " There is something really wrong with me . " Over the past 2 months , the patient has had 4 similar episodes during which she experienced a sudden sinking feeling in her stomach , accompanied by nausea , sweating , tachycardia , and dyspnea . She has called in sick to work 5 times in the last 4 weeks because she fears having an episode at work . The patient has become increasingly fearful of leaving her home . While discussing her symptoms , her breathing suddenly becomes more shallow and rapid , and she starts sweating profusely and feels that her " heart is pounding . " Which of the following is the most important immediate management of this patient's acute symptoms ? A. Buspirone B. Cognitive - behavioral therapy C. Fluoxetine D. Lorazepam E. Propranolol

D. Lorazepam This patient's recurrent episodes of diaphoresis , dyspnea , racing heart , and severe anxiety are due to panic disorder . Panic disorder involves both panic attacks and persistent concern about additional attacks and / or a change in behavior to avoid having another attack . Patients can have a wide array of symptoms , with some coming to the emergency department with fears that they may be very ill ( eg , having a heart attack ) or dying . Long - term treatment of panic disorder may include antidepressants , such as selective serotonin reuptake inhibitors ( SSRIs ) or serotonin - norepinephrine reuptake inhibitors , as well as cognitive - behavioral therapy ; however , both interventions require weeks to take effect and would not alleviate this patient's immediate symptoms and marked distress ( Choices B and C ) . Benzodiazepines eg , lorazepam ) can provide rapid relief of panic attacks and be used while waiting for an SSRI to take effect . However , due to the risks of misuse and addiction , they should be prescribed cautiously in patients with a history of a substance use disorder . Choice A ) Buspirone is an anxiolytic used in the treatment of generalized anxiety disorder . It is not effective in the management of acute anxiety and is not used to treat panic disorder . ( Choice E ) The nonselective beta - blocker propranolol is sometimes used for performance anxiety , but there is no evidence of benefit in panic attacks . Educational objective : Benzodiazepines provide rapid relief of anxiety and are indicated for the management of acutely symptomatic and functionally impaired patients with panic disorder . Antidepressants and cognitive - behavioral therapy are preferred for long - term treatment .

A 16 - year - old boy is brought in for evaluation after an eye injury during a baseball game . The opposing pitcher made a wild throw , and the baseball hit the patient in the right eye . Since then , the patient has been experiencing diplopia which worsens when he looks up . On physical examination , there is mild periorbital ecchymosis and limited vertical movement of the right eye . The remainder of the physical examination is normal . CT scan of the head reveals an orbital floor fracture . Which of the following additional findings is most likely to be seen in this patient ? A. Bruising over the mastoid process B. Impaired olfaction C. Inability to puff out right cheek D. Reduced sensation over the right upper lip and cheek E. Septal hematoma

D. Reduced sensation over the right upper lip and cheek Fractures to the orbital floor , which is composed of the zygomatic bone and maxilla , most commonly result from direct frontal trauma to the orbit due to assault , motor vehicle collisions , or sports injuries . The infraorbital nerve ( continuation of the maxillary nerve ) runs along the orbital surface of the maxilla in the infraorbital groove before traversing the infraorbital canal and exiting the skull via the infraorbital foramen just below the orbit . Damage can result in numbness and paresthesia of the upper cheek , upper lip , and upper gingiva . In addition , displacement of the orbital contents through the floor may cause entrapment of the inferior rectus muscle , resulting in an impaired vertical gaze , and enophthalmos ( posterior displacement of the eyeball within the orbit ) . ( Choice A ) Hematomas over the mastoid process ( Battle sign ) , along with periorbital ecchymosis and clear otorrhea , are signs of basilar skull fractures . ( Choices B and E ) Nasal fractures or high - force trauma to the midface can result in loss of smell if the olfactory nerves are torn from the cribriform plate in the ethmoid bone . Similarly , septal hematomas can occur with impact trauma to the nasal septum but are unlikely in this patient with no nasal trauma evident on physical examination or CT scan . ( Choice C ) The facial nerve ( CN VII ) controls motor function of the face ( ie , ability to puff out cheek ) and can be impaired fractures of the temporal bone after lateral head trauma or zygomatic orbit fractures . Educational objective : Fractures to the orbital floor commonly result from direct frontal trauma to the orbit . The infraorbital nerve runs along the orbital floor . Damage can result in paresthesia of the upper cheek , upper lip , and upper

A 52 - year - old woman comes to the office for hot flashes . In recent months , they have been more intense and more frequent , occurring several times a day , and the patient has been waking up in the middle of the night because her " sheets are drenched in sweat . " Her last menstrual period occurred at age 40 , when she underwent a total abdominal hysterectomy without oophorectomy for uterine fibroids . The patient's medical history is also significant for an unprovoked deep venous thrombosis treated with anticoagulation at age 48. BMI is 24 kg / m² . Which of the following is the best next step in management of this patient ? A. Low - dose clonidine B. Oral estrogen and progestin therapy C. Oral tamoxifen therapy D. Selective serotonin reuptake inhibitor E. Weight loss and vitamin E supplementation

D. Selective serotonin reuptake inhibitor This patient has vasomotor symptoms ( hot flashes ) associated with the menopausal transition . Hot flashes are due to the effects of estrogen withdrawal on the thermoregulatory system in the hypothalamus , which leads to paradoxical peripheral vasodilation , heat loss , and perspiration . Risk factors for hot flashes include obesity , tobacco use , and sedentary lifestyle . Mild hot flashes that do not interfere with daily activities can usually be managed with behavioral modifications such as avoiding spicy foods , dressing in layers , and smoking cessation . Some benefit may also be seen from weight loss in women who are overweight . Vitamin E supplements may be considered , but their benefit is marginal and unlikely to be adequate for this patient's symptoms ( Choice E ) . First - line treatment for moderate to severe vasomotor symptoms of menopause includes estrogen - based hormone replacement therapy ( in patients with an intact uterus , progesterone is added to lower the risk of endometrial hyperplasia and cancer associated with unopposed estrogen ) . However , contraindications to estrogen include a history of thromboembolism , coronary heart disease , breast cancer , and endometrial cancer ( Choice B ) . For patients who are unable to take estrogen , the most effective nonhormonal treatment is a selective serotonin reuptake inhibitor ( eg , fluoxetine ) or serotonin - norepinephrine reuptake inhibitor ( eg . venlafaxine ) . ( Choice A ) Clonidine is a second - line option for women who cannot take estrogen and do not have adequate relief from selective serotonin reuptake inhibitors . However , it provides only a modest benefit and is associated with significant side effects ( eg , dry mouth , constipation , dizziness ) . ( Choice C ) Tamoxifen is a selective estr

A 62 - year - old man presents with a rash and diffuse itching for the past 2 weeks . The rash began on the patient's hands and quickly spread to involve his wrists and upper arms . The itching is worse at night and is only mildly relieved with over - the - counter antihistamines . He has not changed his soap or detergent recently and does not use any hand cream . The patient's spouse had a similar rash a while ago . All vital signs are normal . Skin examination shows small erythematous papules on the palms with burrows and excoriations on the finger webs , palms , and wrist creases . Which of the following is the most appropriate treatment for this patient ? A. Hydrocortisone cream B. Oral doxycycline C. Penicillin G injection D. Topical permethrin E. Topical terbinafine

D. Topical permethrin

A 45 - year - old man is seen about 90 minutes following head trauma . He was trying out his new bicycle when , while attempting to stop . he ran up a curb and fell off , striking the left side of his head . The patient was not wearing a helmet but did not lose consciousness and did not have any observed seizure activity . He has no medical conditions and takes no medications . Vital signs are normal . Modified Glasgow Coma Score is 15. There is an area of swelling and tenderness at the left parietal skull without lacerations or step offs . Pupils are symmetric and react normally , and all extraocular movements are intact . There are no cranial nerve or focal neurologic abnormalities . The remainder of the examination is normal . Which of the following is the best next step in management of this patient ? Discharge with follow - up instruction sheet B. Head CT scan with contrast MRI of the brain Inpatient observat

Discharge with follow - up instruction sheet (Evaluation of mild traumatic brain injury)

A 22 - year - old woman with type 1 diabetes mellitus comes to the emergency department due to sinus congestion . Three weeks ago , she had subjective fever , nasal congestion , and a dry cough that initially improved with over - the - counter medications . However , the patient now has had 2 weeks of purulent nasal discharge , throbbing headache , and facial pain . Temperature is 38 C ( 100.4 F ) . Physical examination shows swollen and erythematous nasal turbinates with yellowish - green nasal secretions . The left maxillary sinus is tender to palpation , with no sinus transillumination . Which of the following is the best next step in management of this patient ? Discharge with continued over - the - counter treatment only Discharge with glucocorticoid nasal spray Discharge with oral antibiotic therapy Hospital admission for intravenous antifungal therapy Order a CT scan of the sinuses

Discharge with oral antibiotic therapy

A 35 - year - old man comes in due to skin sores . Three weeks ago , painful ulcerations developed in the patient's mouth , followed by a diffuse onset of large , painful blisters on the trunk and extremities . Vital signs are normal . On examination , the oral mucosa has several erosions and ulcerations . Scattered , large erosions and a few flaccid bullae are present on the trunk and extremities . Light rubbing of uninvolved skin causes easy separation of the epidermis . Which of the following is the most likely diagnosis in this patient ? A. Bullous impetigo B. Bullous pemphigoid C. Dermatitis - herpetiformis D. Erythema multiforme E. Pemphigus vulgaris

E. Pemphigus vulgaris

A 56 - year - old man comes to the emergency department due to a 2 - day history of fever , chills , shortness of breath , and productive cough . The patient has an episode of vomiting while in the emergency department . Temperature is 38.8 C ( 101.8 F ) , blood pressure is 90/60 mm Hg , and pulse is 102 / min . Leukocyte count is 16,000 / mm³ . Arterial blood gas analysis on room air shows the following : pH 7.49 PaO₂ 70 mm Hg PaCO₂ 30 mm Hg HCO , 24 mEq / L Which of the following is the most likely acid - base status of this patient ? A. Normal - acid - base - balance B. Uncompensated metabolic acidosis uncompensated metabolic alkalosis D. Uncompensated respiratory acidesis E. Uncompensated respiratory alkalesis

E. Uncompensated respiratory alkalesis This patient's presentation is consistent with sepsis due to pneumonia , which can cause a variety of acid - base disturbances . The evaluation of acid - base status follows a stepwise process : 1. Evaluate the pH ( normal : 7.4 ) : This patient's increased pH of 7.49 is indicative of alkalosis ( Choice A ) . 2. Identify the primary process by analyzing the metabolic component of acid - base balance , serum bicarbonate ( HCO3- ) ( normal : 24 mEq / L ) , and the respiratory component , blood PaCO₂ ( normal : 40 mm Hg ) . The parameter that is shifted in the same direction as the pH represents the primary disturbance . This patient's PaCO₂ is low ( shifted toward alkalinity ) , indicating respiratory alkalosis as the primary disturbance . 3. Determine the compensation : Primary metabolic disturbances are quickly compensated via adjustments in alveolar ventilation and CO₂ removal . Compensation for a primary respiratory disturbance begins after several hours via adjustments in renal HCO , excretion and requires several days to complete . Metabolic acidosis is the compensatory response for respiratory alkalosis . This patient's serum bicarbonate of 24 mEq / L indicates that minimal metabolic compensation has taken place . Therefore , this patient has uncompensated respiratory alkalosis , which indicates that hyperventilation due to pneumonia is likely the primary driver of his acid - base status . ( Choice B ) Sepsis is a common cause of primary metabolic acidosis due to lactic acidosis . Unlike in this patient , laboratory results should demonstrate low pH and low HCO3 . ( Choice C ) Vomiting may lead to primary metabolic alkalosis , with laboratory results showing high pH ( as in this patient ) and high HCO , ( not seen in this patient ) . ( Choice D ) Primary respiratory ac

A 68 - year - old man is brought to the emergency department due to confusion and lethargy . The patient lives alone and has type 2 diabetes mellitus and hypertension . Temperature is 36.1 C ( 97 F ) , blood pressure is 99/59 mm Hg , pulse is 132 / min , and respirations are 22 / min . The patient is drowsy and has dry mucous membranes . Neurologic examination shows disorientation to time and place . Laboratory results are as follows : Serum chemistry Sodium 151 mEq / L Potassium 6.1 mEq / L Chloride 121 mEq / L Bicarbonate 18 mEq / L Blood urea nitrogen 42 mg / dL Glucose 810 mg / dL Serum ketones negative ECG shows atrial fibrillation with a rapid ventricular response . Which of the following is the best next step in management of this patient ? A. Correction of hyperkalemia B. High - dose insulin therapy C. Rate control of atrial fibrillation D. Rhythm control of atrial fibrillation E. Volume resuscitation

E. Volume resuscitation

A 64 - year - old man comes to the emergency department with left ankle pain . The patient was playing tennis 2 days ago , attempted to pivot , and felt a sharp pain in his ankle . He has since had difficulty walking and stumbled on the stairs yesterday . On examination , the patient can bear weight , but it is painful . There is diffuse swelling of the ankle . Pulses and sensation in the left foot are normal . Passive range of motion is normal in all directions . Active plantar flexion and dorsiflexion are present . With the patient in the prone position , squeezing the left calf does not cause movement in the left foot . Left ankle x - ray shows soft tissue swelling but no fracture . Which of the following will be most impaired on physical examination ? A. Eversion of the left foot against resistance B. Inversion of the left foot against resistance C. Walking heel to toe D. Walking on the heel of the left foo

E. Walking on tip toes of the left foot This patient who had a sudden , sharp pain in his left ankle while pivoting now has no left foot movement ( ie , passive plantarflexion ) when the examiner squeezes the calf ( ie , simulates gastrocnemius muscle contraction ) . This presentation is most consistent with Achilles tendon rupture . The Achilles tendon connects the gastrocnemius and soleus muscles to the calcaneus and plantar flexes the foot . The diagnosis of acute Achilles tendon rupture may be easily missed because edema or hematoma can mask a palpable tendon defect . The most reliable physical examination maneuver for checking for complete Achilles tendon rupture is the calf squeeze test ( ie , Thompson test ) . With the patient prone and the feet hanging off the table , the provider squeezes the calf , shortening the gastrocnemius muscle . If the foot passively plantar flexes in response , the Achilles tendon is at least partially intact ; the absence of passive plantar flexion indicates complete tendon rupture . Despite Achilles tendon rupture , active plantar flexion can still be present because of the action of accessory muscles of plantar flexion that insert directly onto the foot , rather than the Achilles tendon . These accessory muscles can even maintain some ambulation , although s typically difficult . In general , complete tendon rupture sults an impaired ability to walk on the tips of the toes because this movement requires the power of the gastrocnemius and soleus muscles . ( Choices A and B ) Foot eversion is controlled primarily by the peroneus muscles , which insert mainly onto the lateral aspect of the foot . Foot inversion is controlled by the tibialis anterior and posterior muscles , which insert primarily at the medial aspect of the foot . Neither foot eversion nor inversion would be sign

An 84 - year - old woman with Alzheimer dementia is brought to the emergency department for evaluation after refusing to get out of bed . She has been known to wander the halls at her facility , and a nursing aide reported finding her on the floor next to her bed earlier that day . On examination , the patient is in pain . Her right leg appears shorter than her left . She can wiggle her toes but has significant external rotation of the right lower extremity compared with the left . The lower leg compartments are soft . Which of the following is the most likely diagnosis in this patient ? A. Femoral neck fracture B. Femoral shaft fracture C. Posterior hip dislocation D. Pubic ramus fracture E. Trochanteric fracture

Femoral neck fracture This elderly woman has a right femoral neck fracture presenting with an acutely shortened , externally rotated leg . Femoral neck and intertrochanteric fractures are the most common hip fractures in older adults and most commonly occur due to mechanical falls . Physical examination in femoral neck and intertrochanteric fractures is classically characterized by shortening and external rotation of the leg compared with the contralateral side , which is primarily due to contraction of the psoas and iliacus without the normal acetabular counterforce . This pattern is also seen in anterior hip dislocation , which is significantly less common than fracture and is typically seen following severe trauma ( eg , fall from a height ) . The diagnosis is confirmed with x - ray . Hip fractures are classified as either intracapsular ( femoral head and neck ) or extracapsular ( intertrochanteric or subtrochanteric ) . Intracapsular fractures typically present without significant ecchymoses and have a higher risk of avascular necrosis . Extracapsular fractures are at higher risk for displacement and usually have visible ecchymosis . Both types require surgical correction with either arthroplasty or open reduction with internal fixation . ( Choices B and D ) Femoral shaft fracture in elderly patients can present with shortening of the leg , often with angulation . Pubic ramus fracture can occur from minor ( or no ) trauma in elderly patients and also can cause shortening of the ipsilateral leg . However , external rotation is more typical of femoral neck fracture . ( Choice C ) Posterior hip dislocation typically presents with adduction and internal rotation at the hip . It usually occurs from an axial force on the femur ( eg , dashboard injury ) , and some patients have neurologic manifestations due to invol

Hypokalemia

Flattened or inverted T-wave appearance of U waves Depressed S T segment

A 36 - year - old woman presents due to 12 hours of malaise and fever with chills . The patient also has pain in multiple joints . Her menstrual period started 3 days ago , and she is using 4-6 tampons per day . The patient is sexually active . Temperature is 39.3 C ( 102.7 F ) , blood pressure is 122/81 mm Hg , pulse is 104 / min , and respirations are 14 / min . Examination shows several pustules on the chest , forearms , palms , and fingers . Joint examination shows no swelling , but the right wrist and left ankle are tender on palpation , especially with active or passive joint movement . Heart sounds are normal . Funduscopic and oropharyngeal examinations are normal . There is no cervical , axillary , or inguinal lymphadenopathy . Which of the following is the most likely diagnosis ? Gonococcal infection Parvovirus B19 Rheumatic fever Secondary syphilis Staphylococcal toxic shock syndrome

Gonococcal infection This patient likely has disseminated gonococcal infection ( DGI ) , which typically presents with purulent arthritis or the following triad : 1. Polyarthralgia : Asymmetric pain in multiple distal and proximal joints . Examination usually reveals pain with movement and palpation ; multiarticular joint swelling , erythema , and warmth are uncommon . 2. Pustular rash : Most patients have 2-10 pustular or vesiculopustular lesions on the distal extremities ; trunk lesions can also occur . The palms and soles may or may not be affected . 3. Tenosynovitis : Patients report pain over the flexor tendons of multiple distal joints ( eg , wrists , ankles , fingers , toes ) and / or pain with passive range of motion of the joint . Systemic signs of infection ( eg , fever , chills , malaise ) are frequently present . Although examination ( eg , mucopurulent discharge ) or testing ( nucleic acid amplification ) often reveals urogenital infection , many patients have no urogenital symptoms The diagnosis of DGI requires cultures of blood and synovial fluid ( when joint effusion present ) and nucleic acid amplification testing NAAT ) of samples from mucosal sites ( eg , urethra , throat , rectum ) . Because Neisseria gonorrhoeae grows fastidiously , blood and synovial fluid cultures are frequently negative ( fewer than one - third of patients had positive blood cultures in one study ) ; therefore , the diagnosis is usually presumptively established by NAAT of urogenital sites .

Pemphigus vulgaris High -Treatment

High - dose corticosteroids Local wound care

A 28 - year - old woman comes to the emergency department due to severe right lower quadrant pain . Yesterday , the patient noticed intermittent right lower abdominal pain associated with increased activity . She rested the remainder of the day , but the pain continued to increase . An hour ago , the pain suddenly became constant , severe , and associated with nausea and vomiting . The pain does not radiate . Temperature is 37.2 C ( 99 F ) , blood pressure is 130/80 mm Hg , and pulse is 98 / min . There is tenderness to palpation over the right lower quadrant . Urine pregnancy test is negative . Which of the following additional information would be most helpful in establishing this patient's diagnosis ? A. Associated urinary symptoms B. Family history of malignancy C. History of ovarian cysts D. Number of sexual partners E. Recent sick contacts

History of ovarian cysts

A 15 - year - old girl is seen after falling in the shower . The patient states she slipped and denies any loss of consciousness , tongue biting , or confusion . Her parents are concerned that she fell because she is thin and eats poorly . The patient admits she has felt a " little down " since her boyfriend broke up with her , but she does not blame him because she is " so fat . " Blood pressure is 100/60 mm Hg sitting and 80/48 mm Hg standing , and pulse is 50 / min . BMI is 16 kg / m² . Examination reveals dry mucous membranes and emaciation . Calluses are present over the knuckles . The abdomen is soft , nontender , and nondistended . Serum potassium level is 2.8 mEq / L . Which of the following is the best next step in management of this patient ? Ambulatory 24 - hour Holter monitoring Cognitive - behavioral therapy Hospitalization Nutritional rehabilitation Trial of fluoxetine

Hospitalization This patient's low body weight ( BMI ≤18.5 kg / m² ) , distorted body image , and evidence of self - induced vomiting ( knuckle calluses ) are consistent with anorexia nervosa ( AN ) , purging subtype . AN occurs most commonly in adolescent girls , particularly those with perfectionist personalities from affluent families or those participating in activities in which there is pressure to be thin ( eg , gymnastics , ballet ) . Although AN can improve with outpatient treatment programs , hemodynamic instability requires hospital admission and inpatient treatment . Self - induced vomiting can cause severe dehydration , resulting in light - headedness or syncope , a blood urea nitrogen / creatinine ratio > 20 : 1 , and orthostatic hypotension ( decrease in systolic blood pressure of 220 mm Hg and in diastolic blood pressure of 210 mm Hg with standing ) . Other indications for hospitalization include arrhythmia , severe bradycardia , acute food refusal , electrolyte abnormalities ( eg , hypokalemia ) , and suicidality . Once the patient is medically stabilized , the mainstay of treatment for AN is psychotherapy and nutritional rehabilitation ( Choices B and D ) . ( Choice A ) Referral for ambulatory ( outpatient ) Holter monitoring is inappropriate because hemodynamically unstable patients require admission . Inpatient telemetry is indicated for monitoring of arrhythmias in patients with bradycardia or electrolyte derangements . ( Choice E ) Fluoxetine is a selective serotonin reuptake inhibitor that is useful for depression and bulimia but not AN . Although this patient should be assessed further for comorbid depression , antidepressants would not treat her AN . Educational objective : The mainstay of treatment in anorexia nervosa is psychotherapy and nutritional rehabilitation . Indications for hospita

A 63 - year - old man is brought to the emergency department due to progressive upper and lower extremity weakness . Today , the patient was unable to get out of bed . Medical history is significant for liver cirrhosis and portal hypertension ; medications include nadolol and spironolactone . Blood pressure is 90/60 mm Hg and pulse is 70 / min . ECG is shown in the exhibit . Which of the following electrolyte abnormalities is the most likely cause of this patient's clinical presentation ? Hypercalcemia Hyperkalemia Hypocalcemia Hypokalemia Hyponatremia

Hyperkalemia This patient with cirrhosis has characteristic ECG changes suggesting hyperkalemia as the cause of his weakness . Causes of hyperkalemia can include beta - blocker therapy and use of potassium - sparing diuretics ( eg , spironolactone ) . Elevated extracellular potassium increases the membrane excitability at the neuromuscular junction . This eventually leads to prolonged depolarization , resulting in inactivation of the sodium channels and reduced ability to fire an action potential ( impaired neuromuscular transmission ) . Hyperkalemia also interferes with repolarization of the cardiac action potential . Manifestations of hyperkalemia include diffuse muscle weakness and cardiac conduction abnormalities . The earliest ECG change in hyperkalemia is T waves with high amplitudes and narrow bases ( peaked T - waves ) ; further abnormalities include the loss of the P wave , prolonged PR interval , widening of the QRS complex , and slowed heart rate . In severe cases , hyperkalemia may progress to a sine - wave pattern , which manifests clinically as ventricular fibrillation or asystole . ( Choice A ) Hypercalcemia can cause muscle weakness but commonly presents with constipation , nausea , and vomiting . ECG changes consistent with hypercalcemia include QT - interval shortening ; severe cases may cause T - wave flattening or inversion . ( Choice C ) Hypocalcemia causes QT - interval prolongation . Symptoms include paresthesia , spasms , and muscle twitching , not muscle weakness . (Choice D ) Hypokalemia can present like hyperkalemia , with muscle weakness and muscle cramps . However , hypokalemia causes . QT - interval prolongation , the appearance of U waves , and ST - segment depression . Severe hypokalemia may cause torsade de pointes . ( Choice E ) Hyponatremia is not classically associated with speci

A 75 - year - old man is brought to the emergency department due to 2 days of progressive lethargy and confusion . He has had fatigue and blurred vision over the last several days . The patient was recently diagnosed with polymyalgia rheumatica and is taking prednisone . He also has diet - controlled type 2 diabetes mellitus . Temperature is 37.9 C ( 100.2 F ) , blood pressure is 90/60 mm Hg , pulse is 112 / min , and respirations are 18 / min . Oral mucous membranes are dry . The patient is somnolent but arousable and moves all extremities equally on command . Laboratory results are as follows : Sodium 129 mEq / L Potassium 3.5 mEq / L Chloride 94 mEq / L Bicarbonate 24 mEq / L Blood urea nitrogen 36 mg / dL Glucose 1,170 mg / dL Urinalysis shows trace ketones . Which of the following is the most likely cause of this patient's altered mental status ? Glucocorticoid psychosis Hyperosmolality Hyponatremia

Hyperosmolality

A 17 - year - old girl comes to the emergency department due to painful menses . The patient's last menstrual period began 2 days ago ; however , she has had 3 days of severe , crampy lower abdominal pain along with nausea and headaches . The pain has been unresponsive to acetaminophen . The patient has no change in bowel pattern , vaginal discharge , or irregular menstrual bleeding . She has monthly menses with 4 days of moderate bleeding with severe , painful cramping , often requiring her to miss school . The patient is not sexually active . Vital signs are normal . The abdomen is soft and nontender . The uterus is small and nontender with no adnexal masses . There is no abnormal cervical discharge . Which of the following is the best next step in evaluation of this patient ? A. CT scan of the abdomen and pelvis ( 1 % ) B. Diagnostic laparoscopy ( 4 % ) C. No further diagnostic testing ( 55 % ) D. Nucleic aci

In adolescents , primary dysmenorrhea typically begins with the establishment of ovulatory cycles ( eg , regular monthly menses ) . In these patients , excessive local prostaglandin production causes inflammation and painful uterine contractions ( eg , cramping ) during menstruation that can interfere with daily activities . The elevated prostaglandin levels also cause associated gastrointestinal tract symptoms ( eg , nausea , vomiting , diarrhea ) . These symptoms begin just prior to the onset of menstruation and generally last 2-3 days . Risk factors include heavy menstrual periods , menarche at age < 12 , and sexual abuse . Primary dysmenorrhea is a clinical diagnosis in patients with cyclic pelvic pain during menses and a normal pelvic examination ( eg , no fixed uterus or adnexal mass ) , suggesting no underlying pathology . Therefore , no additional diagnostic testing is indicated , and empiric treatment with nonsteroidal anti - inflammatory medications and / or combination oral contraceptives can be started . Primary dysmenorrhea typically improves with advancing age and frequently resolves after childbirth . Choices A , B , and E ) Evaluation via CT scan , laparoscopy , or ultrasound is indicated medical management of primary dysmenorrhea fails or when secondary dysmenorrhea ( pelvic pain caused by an underlying pathology ) is suspected . Secondary dysmenorrhea is more likely to present with noncyclic pelvic pain , dyspareunia , abnormal uterine bleeding , and abnormal examination findings ( eg , enlarged or immobile uterus , adnexal mass ) . Choice D ) Nucleic acid amplification testing evaluates for acute cervicitis from Chlamydia chomatis and eria gonorrhoeae . Acute cervicitis can present with lower abdominal pain but is unlikely in a patient who is not sexually active . Educational objective : Primary dysm

Ovarian torsion Summary

In nonpregnant women , acute lower abdominal pain can have various causes involving the gastrointestinal , gynecologic , or urologic systems . Many of these causes have classic presenting symptoms that characterize the disease process and differentiate the diagnosis . This patient's right lower quadrant pain is classic for ovarian torsion , which occurs due to rotation of the ovary around the infundibulopelvic ligament , causing ovarian vessel occlusion . Although ovarian torsion can occur in all reproductive - aged women , those with a history of ovarian cysts or masses are at increased risk because the greater size and density of the ovary make it prone to rotation and subsequent torsion . Patients initially have intermittent pain associated with activity due to partial ovarian torsion ; the pain resolves when the adnexa spontaneously untwists and blood flow returns . When this progresses to complete ovarian torsion , patients typically develop sudden - onset , severe , nonradiating pain ( due to persistent ischemia ) often associated with nausea and vomiting .

A 46 - year - old woman is brought to the emergency department following a ground - level fall at home . She slipped while getting out of the shower and hit her left knee on the tile floor . The patient developed severe left knee pain and swelling and is unable to ambulate . Examination shows a contusion and soft tissue swelling at the knee with a large knee effusion . X - ray of the knee is shown in the image above . Which of the following examination findings would most likely be seen in this patient ? Backward movement of the tibia against a fixed femur Excessive joint widening on valgus stress Forward movement of the tibia against a fixed femur Inability to extend the knee against gravity Inability to flex the knee against resistance

Inability to extend the knee against gravity This patient has a comminuted patella fracture . The patella is a large sesamoid bone that facilitates knee extension , protects the joint from direct injury , and improves nourishment of the distal femur articular cartilage . The quadriceps tendon inserts at the superior pole of the patella and wraps distally around the patella to become the patellar tendon , which inserts at the tibial tuberosity . Patella fractures are most commonly due to a direct blow to the anterior aspect of the knee ( eg , landing on a flexed knee in a fall , striking the knee against the dashboard in a motor vehicle collision ) . The patella can also be fractured indirectly due to excessive force transmitted through the quadriceps tendon ( eg , landing on the feet after falling from a height ) . Patients develop acute swelling ( often with an associated effusion ) , focal tenderness and inability to extend the knee against gravity ( as the quadriceps muscle cannot contract ) .

A 58 - year - old man with Burkitt lymphoma comes to the emergency department for nausea and generalized weakness . He started chemotherapy 2 days ago . His other medical conditions include hypertension , stable angina , and hyperlipidemia . Serum chemistry results are as follows Sodium 140 mEq / L Potassium 6.8 mEq / L Bicarbonate 18 mEq / L Blood urea nitrogen 24 mg / dL Creatinine 1.6 mg / dL ECG shows peaked T waves . Which of the following interventions should be used next to rapidly lower serum potassium in this patient ? Furosemide ( 10 % ) Hemodialysis ( 11 % ) Insulin and glucose ( 63 % ) Normal saline % ) Sodium polystyrene sulfonate ( 4 % )

Insulin and glucose ( 63 % ) Normal saline

A 12 - year - old girl is brought to the emergency department due to rash and difficulty breathing . Symptoms started acutely following a bee sting ; the patient developed full - body hives , difficulty breathing , and vomiting immediately . On arrival , blood pressure is 110/70 mm Hg , pulse is 116 / min , and respirations are 28 / min . Pulse oximetry is 91 % on room air . Examination shows a girl in moderate respiratory distress with truncal hives and diffuse wheezing heard on auscultation . Intramuscular epinephrine is administered , with resolution of symptoms and normalization of vital signs . However , an hour later , she has worsening hives , mild wheezing , and an additional episode of emesis . Repeat respiratory rate is 18 / min and pulse oximetry is 97 % . Which of the following is the best next step in management of this patient ? XOA . Emergency intubation B. Inhaled albuterol C. Intramuscular epinep

Intramuscular epinephrine This patient with recurrence of anaphylactic symptoms ( eg , hives , wheezing , emesis ) after initial resolution is having a biphasic anaphylactic reaction and should be treated with an additional dose of intramuscular ( IM ) epinephrine . Anaphylaxis is an acute , life - threatening , IgE - mediated , type I hypersensitivity reaction characterized by allergic symptoms affecting 22 organ systems or sudden hypotension after allergen exposure ( eg , bee sting ) . It often resolves quickly and completely after epinephrine administration ( ie , uniphasic anaphylaxis ) . However , symptoms may recur or persist despite treatment , according to the following patterns : • Biphasic anaphylaxis , as seen in this patient , is characterized by an initial anaphylactic reaction followed by an asymptomatic period typically 1-30 hr ) and then a recurrence of symptoms . • Protracted anaphylaxis ( less common ) is characterized by anaphylactic symptoms that last hours or days despite treatment Patients with biphasic or protracted anaphylaxis ( or those unresponsive to the initial dose ) should receive additional IM epinephrine . If symptoms persist after 3 IM doses , intravenous epinephrine infusion may be required . Adjunct therapies may target specific symptoms of anaphylaxis ( eg , inhaled albuterol for bronchospasm , antihistamines for hives ) . However , only epinephrine , which has both beta - 2 agonist ( eg , bronchodilation , release of inflammatory mediators ) and alpha - 1 agonist ( eg , vasoconstriction , upper airway edema ) effects , can treat all components of anaphylaxis . Therefore , it takes priority over adjuncts ( Choice B ) . Choice A ) Patients with anaphylaxis and prominent upper airway edema ( eg , oropharyngeal swelling , stridor , voice alteration ) may require endotracheal intuba

A 47 - year - old man is brought to the emergency department due to lightheadedness and palpitations . He reports a history of palpitations since he was a teenager and says he can usually stop them by bearing down . However , this maneuver did not work this time , so he decided to come to the hospital . His other medical problems include type 1 diabetes mellitus and hypertension . He had a myocardial infarction 5 years ago , but has not had chest pain since then . His exercise tolerance is good . Blood pressure is 80/40 mm Hg and his pulse is 150 / min . Lungs are clear on auscultation . No heart murmurs are appreciated . ECG is shown in the exhibit . Which of the following is the best next step in managing this patient ? Bedside transthoracic echocardiogram Carotid sinus massage Intravenous adenosine Intravenous amiodarone Intravenous sedation and cardioversion

Intravenous sedation and cardioversion This patient with palpitations has an ECG showing a wide - complex tachycardia , consistent with ventricular tachycardia ( VT ) , that has lead to hemodynamic instability ( eg , hypotension ) . Based on Advanced Cardiac Life Support ( ACLS ) guidelines , patients with any persistent tachyarrhythmia causing clinical ( eg , chest pain ) and / or hemodynamic instability should have immediate synchronized cardioversion . Adequate sedation and analgesia are required prior to cardioversion . If the patient is hemodynamically stable , attempts should be made to differentiate between wide - complex tachycardia ( QRS > 120 milliseconds ) and narrow - complex tachycardia ( QRS ≤120 milliseconds ) . Patients with narrow - complex tachycardia likely have supraventricular tachycardia ( SVT ) and can be treated accordingly ( eg , adenosine ) . Educational objective : Immediate synchronized cardioversion is the initial treatment of choice in patients with wide - complex tachycardia associated with clinical and / or hemodynamic instability

A 54 - year - old homeless man is brought in by police after he was found confused and disheveled . Temperature is 36.5 C ( 97.7 F ) . The patient is thin , appears ill , and has dry mucous membranes . The abdomen is nondistended . The patient is disoriented and unable to recognize where he is or state the date . Bilateral pupils are equal and reactive to light , but extraocular motion is restricted on leftward gaze . Deep tendon reflexes are reduced . The patient's gait is markedly ataxic . Laboratory results are as follows : Hemoglobin 9.6 g / dL Mean corpuscular volume 104 μm³ Albumin 2.2 g / dL Aspartate aminotransferase ( SGOT ) 112 U / L Alanine aminotransferase ( SPGT ) 52 U / L Which of the following is the best next step in management of this patient's confusion ? Intravenous broad - spectrum antibiotics Intravenous crystalloid and dextrose Intravenous lorazepam Intravenous thiamine Oral lactul

Intravenous thiamine

A 22 - year - old woman has had a severe left frontal headache for the past 30 minutes . The patient has had similar pain every day for the past 3 days , and the episodes resolve spontaneously after about an hour . She has no other medical conditions and takes no medications . On physical examination , the patient appears restless and in pain . There is conjunctival injection and lacrimation on the left side . Which of the following findings is most suggestive of cluster headache in this patient ? Episodic nature Female sex Frontal location Ipsilateral conjunctival injection Unilateral pain

Ipsilateral conjunctival injection

Ovarian torsion Treatment

Laparoscopy with detorsion • Ovarian cystectomy • Oophorectomy if necrosis or malignancy

A 51 - year - old man is brought to the emergency department due to chest tightness that started 30 minutes prior to arrival . His chest discomfort is associated with shortness of breath and nausea . Initial ECG shows ST - segment elevation in leads I and aVL . Cardiac enzymes are elevated . This patient most likely has occlusion of which of the following arteries ? Distal left anterior descending artery Left circumflex artery Left main coronary artery Proximal left anterior descending artery Right coronary artery

Left circumflex artery

A 42 - year - old man comes to the office due to back pain . Yesterday , the patient was moving furniture when he suddenly had sharp , shooting pain in the lower back and buttocks that radiated down the back of the legs . Today , his legs felt numb and weak , and he also had to strain to urinate . Vital signs are normal . Physical examination shows lower lumbar spine tenderness on palpation . Bilateral lower extremity strength is 4/5 , and ankle reflexes are absent bilaterally . Sensation to touch is decreased in the perineal region and buttocks . Straight leg raise testing is positive . Which of the following is the best next step in management of this patient ? MR imaging of the lumbosacral spine Nonsteroidal anti - inflammatory drugs and resume activity as tolerated Plain radiographs of the lumbosacral spine Physical therapy for graduated exercises Strict bed rest and lumbar support

MR imaging of the lumbosacral spine Features of cauda equina syndrome

A 2 - year - old boy is brought to the emergency department due to worsening right ear pain . The patient has been tugging at his ear for 3 days and has had a fever . He has been less active but has had no vomiting , diarrhea , or rash . Temperature is 40 C ( 104 F ) . Examination shows a tired , irritable boy who screams with any attempt to examine his ear . Swelling , erythema , and tenderness to palpation are noted posterior to the right ear . Otoscopy reveals an erythematous and bulging tympanic membrane with loss of normal landmarks and light reflex . CT scan of the temporal bones reveals fluid in the mastoid air cells with erosion of bony septa . Which of the following is the best next step in management of this patient ? A. Lumbar puncture and cerebrospinal fluid culture B. Middle ear drainage and IV antibiotics C. Oral antibioties and follow - up CT sean in a week D. Ototopical antibiotics and glucocorti

Middle ear drainage and IV antibiotics This febrile child with a bulging tympanic membrane who has mastoid tenderness and displacement of the external ear likely has acute mastoiditis , the most common suppurative complication of acute otitis media ( AOM ) . The mastoid air cells are in continuity with the middle ear , allowing spread of the infection . In addition to the symptoms of fever and ear pain seen in AOM , infection of the mastoid causes erythema and tender swelling , which can result in outward , vertical displacement of the auricle . The tympanic membrane may show evidence of AOM ( eg , erythema , bulging , perforation ) or may not be visible due to swelling of the external auditory canal . Acute mastoiditis is often diagnosed clinically ; however , imaging ( eg , CT scan ) is indicated if the diagnosis is unclear , if there is suspicion for further complications ( eg , meningitis , neurologic deficits ) , or if the child appears toxic . In these cases , CT scan reveals destruction of the bony septa in the mastoid air cells , confirming the diagnosis . Treatment is with intravenous antibiotic therapy with activity against Streptococcus pneumoniae , Streptococcus pyogenes , and Staphylococcus aureus . Expanded coverage for Pseudomonas should be used if there is recent antibiotic use . Drainage of the purulent material is also required with either tympanostomy ( ± ear tube placement ) or mastoidectomy .

A 9 - year - old girl with a history of poorly controlled , moderate , persistent asthma is brought to the emergency department with wheezing and respiratory distress . She also has had 3 days of rhinorrhea , cough , and low - grade fevers . Temperature is 37.2 C ( 99 F ) , blood pressure is 100/60 mm Hg , pulse is 128 / min , and respirations are 32 / min . Pulse oximetry is 89 % on room air . Examination reveals significantly decreased aeration , diffuse expiratory and inspiratory wheezes , and subcostal retractions . Which of the following most likely decreases this patient's immediate risk of hospitalization ? A. Inhaled fluticasone B. Inhaled salmeterol and albuterol C. Nebulized albuterol and ipratropium D. Nebulized albuterol E. Oral corticosteroids

Nebulized albuterol and ipratropium This patient has a severe asthma exacerbation characterized by tachypnea , increased work of breathing , hypoxia , and diffuse wheezing . Inhaled or nebulized short - acting beta agonists ( SABAs ) are the first - line treatment for asthma exacerbations as they provide rapid bronchodilation . Albuterol is the most commonly used SABA . In moderate to severe exacerbations , inhaled or nebulized anticholinergics ( eg , ipratropium ) are also administered for further bronchodilation . Randomized controlled studies in children comparing the combination of inhaled anticholinergics and SABA to SABA alone have demonstrated that combination therapy lowers the risk of hospital admission ( Choice D ) . In addition , initial treatment in the emergency department for moderate to severe exacerbations includes systemic corticosteroids to decrease airway edema and secretions . Choice A ) Inhaled corticosteroids ( ICS ) ( eg , fluticasone ) are chronic controller medications for patients with persistent asthma . Although poor adherence to ICS can increase the risk of asthma exacerbations , inhaled ICS have a slower onset of action compared to systemic corticosteroids . Administration in the emergency department has not been shown to decrease hospitalization risk . ( Choice B ) Salmeterol is a long - acting beta agonist used as chronic therapy in patients with moderate or severe persistent asthma . Long - acting beta agonists are not used in the acute treatment of an asthma exacerbation . Choice E ) Systemic corticosteroids help reduce airway inflammation within 2-4 hours of administration and can reduce the likelihood of admission . However , inhaled or nebulized SABAs and anticholinergics provide immediate bronchodilation and are the mainstay of treatment . Educational objective : Initial tre

A 44 - year - old man presents with sudden right foot pain that developed after pivoting off the ground while playing basketball . He immediately fell to the ground and has not been able to walk without assistance since . On examination , the lateral aspect of the right foot anterior to the ankle is tender to palpation and has significant swelling and bruising . There is no pain over the medial or lateral malleoli . Pulses are normal . X - ray of the foot shows a fifth metatarsal proximal diaphysis fracture as shown above . Which of the following is the best next step in management of this patient ? Bone mineral density testing Elastic wrap and weight - bearing as tolerated Immediate surgical fixation MRI of the ankle and foot Non - weight - bearing short leg cast

Non - weight - bearing short leg cast

A 64 - year - old woman is evaluated for sudden - onset headache with nausea , vomiting , and a sensation that the " room is spinning . " The patient developed symptoms yesterday when she awoke , and they have not improved . She has had no ear pain or hearing loss . Medical history includes hypertension , type 2 diabetes mellitus , and hyperlipidemia . Blood pressure is 124/76 mm Hg and pulse is 86 / min . Neurologic examination with the patient lying still shows nonfatigable nystagmus . Walking worsens symptoms , and the patient falls toward the left side . Which of the following is the best next step in management of this patient ? A. Ambulatory EGG monitoring B. Epley canalith repositioning maneuver C. Noncentrast CT sean of the head D. Perform lumbar puncture E. Reassurance and oral meclizine therapy

Noncentrast CT sean of the head This patient with vascular risk factors , acute - onset headache , a sensation that the room is spinning , and nystagmus likely has had a stroke or hemorrhage affecting the cerebellum . Vertigo is often described as a sensation of spinning ( as in this patient ) , whirling , or tilting and can be due to an underlying disease in either the central or the peripheral nervous system . Patients with central vertigo have nystagmus that is not fatigable and not inhibited by fixation of gaze . They may be unable to walk because , in addition to a false sensation of movement , vertigo from a central underlying cause ( eg , stroke ) may also lead to postural instability and lack of coordination . Patients with central vertigo , such as this patient , should be evaluated for cerebellar stroke or hemorrhage , particularly if they have the following : • Prominent stroke risk factors ( eg , hyperlipidemia , hypertension , diabetes mellitus ) New - onset headache • Neurologic signs / symptoms ( eg , weakness / numbness of the face or limbs , dysarthria ) Head imaging should be obtained urgently . CT scan is often obtained first because it can be completed faster than an MRI . However , if the CT scan is negative , an MRI is usually required to exclude brainstem or cerebellum pathology , which may not be visualized well on CT scan . Choice A Ambulatory ECG monitoring can be used to diagnose underlying cardiac arrhythmias or conduction abnormalities . However , the " dizziness " at presentation is typically presyncope or syncope rather than vertigo . Choice B Canalith repositioning maneuvers ( eg , Epley ) are used to treat benign paroxysmal positional vertigo ( BPPV ) . In contrast to this patient's condition , BPPV does not cause headache but affects the semicircular canals and therefore has cha

A 52 - year - old woman comes to the emergency department ( ED ) on a Saturday night with severe low back pain . She has a history of chronic back pain , for which her pain - management physician prescribes oxycodone . However , the patient's pain has become worse recently , and her normal dose of oxycodone has not been effective . She is very angry because she called her physician's office multiple times before finally reaching the covering physician , who directed her to go to the ED . The patient requests that the emergency provider prescribe a stronger pain medication so that " I can get some relief . " She has no prior history of visits to this ED , and no medical records are available . Which of the following is the most appropriate reaction to the patient's request ? Defer treatment and instruct her to call her outpatient physician on Monday Explain the risks of opiate addiction and recommend nonnarcotic pa

Obtain additional information about the patient's pain problem

A 46 - year - old woman with type 2 diabetes mellitus comes to the office for evaluation of a rash on her breast . For the past 2 months , the patient has had a pruritic rash on her left breast that has slowly spread . She has had no fever , nipple discharge , or recent changes in soaps or laundry detergent . The patient has used a topical antibiotic ointment and a drying powder over the rash but has had no improvement . BMI is 37 kg / m² . The left breast is diffusely enlarged with an area of erythema below the areola that extends to the inframammary fold . There are no palpable breast masses . Two nontender lymph nodes are palpated in the left axilla . The remainder of the examination is unremarkable . Which of the following is the best next step in management of this patient ? A. Biopsy of the breast lesion B. KOH skin scraping C. MRI of the breast D. Oral - antibioties E . Topical steroids

Oral estrogen and progestin therapy

Patellar Fracture

Patella fractures are most commonly due to a direct impact to the anterior aspect of the knee . Signs include an acutely swollen knee , focal patella tenderness , and an inability to extend the knee against gravity .

A 1 - week - old girl is brought to the emergency department by ambulance due to fever and irritability . The patient's mother and father are both age 16 , and the father is not involved in the infant's care . The patient lives with the mother and maternal grandparents in their home . The mother is the only family member present in the emergency department . Temperature is 38.3 C ( 101 F ) and pulse oximetry is 87 % on room air . The patient appears cyanotic . Anterior and posterior fontanelles are bulging . Pulmonary examination reveals tachypnea and intercostal retractions . The examination is concerning for meningitis , and the patient requires intubation and mechanical ventilation ; however , the mother refuses the procedure . Which of the following is the best next step in management of this patient ? Attempt to contact infant's father for consent Call local child protective services Contact hospital legal c

Proceed with mechanical ventilation without consent

A 56 - year - old woman presents with shortness of breath , which began suddenly 2 hours ago while she was watching television . The patient also reports sharp , left - sided chest pain that worsens with cough . Medical history includes diabetes mellitus , hypertension , and a 30 - pack - year smoking history . Temperature is 38.1 C ( 100.5 F ) , blood pressure is 110/60 mm Hg , pulse is 140 / min and irregular , and respirations are 30 / min . Oxygen saturation is 84 % on room air . The patient is in acute distress and diaphoretic . Jugular venous pressure is slightly elevated . The lungs are clear to auscultation . Chest x - ray is unremarkable . ECG reveals irregular RR intervals with no definite P waves and narrow QRS complexes . Which of the following is the most likely diagnosis ? A. Cardiac tamponade B. Mitral stenosis C. Myocardial infarction D. Pulmonary embolism E. Tension pneumothorax

Pulmonary embolism The manifestations of pulmonary embolism ( PE ) are nonspecific and variable . Acute - onset dyspnea and pleuritic chest pain are the most common symptoms , followed by hemoptysis and symptoms of deep venous thrombosis . This patient has multiple physical examination findings consistent with PE , including tachypnea , tachycardia , hypoxemia , low - grade fever , and jugular venous distension . She does not have the classic findings of PE on ECG ( ie , prominent S in lead I , Q in lead III , and inverted T in lead III [ S1Q3T3 ] ) or chest x - ray ( eg , Hampton hump , Westermark sign ) , but these findings are present in only a minority of cases . Atrial fibrillation ( irregular RR intervals with narrow QRS complexes and an absence of organized P waves ) is commonly associated with PE . In some cases , the atrial fibrillation may be the cause ( rather than result ) of PE due to embolization of thrombus from the right atrium into the pulmonary circulation . Hypoxemia and atrial fibrillation are both associated with poor prognosis in PE . ( Choice A ) Cardiac tamponade may present with dyspnea , tachypnea , and tachycardia . However , it is less likely to cause sharp chest pain and hypoxemia , and this patient's lack of distant heart sounds and hypotension make this diagnosis less likely . ( Choice B ) Mitral stenosis is a common cause of atrial fibrillation resulting from left atrial dilation . However , for mitral stenosis to cause abrupt - onset shortness of breath , pulmonary edema is expected on chest x - ray . ( Choice C ) This patient has risk factors for myocardial infarction ( eg , hypertension , diabetes mellitus ) ; however , pleuritic chest pain , hypoxemia without pulmonary edema , of ischemic ECG changes ( eg , ST elevation , inversion , Q waves ) make more likely . ( Choice E ) Tensio

A 52 - year - old man comes in an hour after the onset of severe substernal chest pressure . The pressure started when he was helping his brother who has coronary artery disease , move into a new house and was associated with mild dizziness and shortness of breath . He took his brother's nitroglycerin tablets and , after 45 minutes , the pain subsided . The patient has hypertension and hyperlipidemia . Blood pressure is 152/93 mm Hg and pulse is 105 / min . ECG shows sinus tachycardia and left ventricular hypertrophy , similar to his ECG 6 months ago . Troponin level is normal . Which of the following is the best next step in management of this patient A. Gardiae catheterization B. Discharge with close follow - up C. Exercise radionuclide perfusion scan D. Intravenous alteplase Repeat ECGs and troponin levels

Repeat ECGs and troponin levels This patient has several cardiovascular risk factors ( eg , hypertension , hyperlipidemia , family history ) and developed typical anginal pain during exertion . Stable anginal pain usually resolves within a few minutes of rest or sublingual nitroglycerin administration ; anginal pain lasting longer than 20 minutes is suggestive of acute coronary syndrome ( ACS ) . Initial ECG may be nondiagnostic or normal in more than half of patients with myocardial infarction ( MI ) , and serum troponin levels can remain undetectable for 6-12 hours following the onset of MI symptoms . For patients with initial negative ECGS and troponin levels but a reasonable suspicion for ACS , management is with further observation and serial ECGS and troponin levels ( eg , several ECGS 30 min apart , 3 troponin levels 6 hr apart ) . Subsequent elevation in serum troponin levels with or without ECG findings of ischemia ( eg , ST - segment depression , T - wave inversions with prominent R waves ) confirms non - ST - segment elevation MI ( STEMI ) . However , reoccurrence of typical anginal pain in the absence of serum troponin elevation may confirm unstable angina . Choice A ) Emergency cardiac catheterization with revascularization is indicated in patients with evidence of STEMI on initial ECG or ACS with hemodynamic instability . Cardiac catheterization is also indicated within 24-48 hours in patients with non - STEMI , but it should not be performed in this patient until a diagnosis is established . ( Choice B ) Reassurance and discharge with close follow - up may eventually be appropriate in this patient but not without further observation and serial ECG and troponin levels to rule out ACS . ( Choice C ) An exercise radionuclide perfusion scan allows for the identification of myocardial regions that have i

A 65 - year - old woman presents with vision disturbances in her left eye . She underwent cataract extraction with intraocular lens implantation in the left eye 4 months ago . The surgery was uncomplicated , and the patient experienced no vision issues until this morning , when she suddenly noticed " dark spots " in her peripheral vision and bright flashes of light with left eye movement . The patient has had no eye pain or trauma , headache , fever , or chills . Vital signs are normal . Left eye examination shows clear conjunctiva and cornea and decreased peripheral vision in the temporal visual field . The right eye is normal . Which of the following is the most likely diagnosis ? Anterior uveitis Central retinal artery occlusion Intraocular lens dislocation Optic neuritis Retinal detachment

Retinal detachment This patient , who is experiencing bright flashes of light and an isolated defect in the left eye peripheral vision , most likely has retinal detachment , which occurs when there is separation of the retinal neurosensory layer from the underlying retinal pigment epithelium and choroid . Retinal detachment may be a complication ( acute or delayed ) of intraocular surgery ; other common risk factors include advanced age , myopia , and ocular trauma .

A 29 - year - old woman is brought to the emergency department after a syncopal episode while getting out of bed . Yesterday , she developed right - sided abdominal pain that has spread across the lower abdomen over the past few hours . The patient has ulcerative colitis and is on mesalamine therapy . Her last menstrual period was 8 weeks ago . Temperature is 37 C ( 98 F ) , blood pressure is 90/60 mm Hg , and pulse is 125 / min . She has diffuse lower abdominal pain with rebound tenderness and voluntary guarding . Pelvic examination shows cervical motion tenderness and right - sided adnexal tenderness ; no masses are palpated . Which of the following is the most likely diagnosis for this patient ? A. Appendicitis B. Colonic perforation C. Ovarian torsion D. Pelvic inflammatory disease E. Ruptured ectopic pregnancy

Ruptured ectopic pregnancy

Dermatitis herpetiformis

Seen in patients with celiac disease

A 66 - year - old man is evaluated for worsening abdominal pain . Over the past week , he has had left lower quadrant discomfort , nausea , and anorexia but no dysuria or hematuria . This morning , the patient had sudden , severe lower quadrant pain accompanied by an episode of vomiting . The pain initially improved but then gradually intensified to involve the entire abdomen . Temperature is 38.3 C ( 100.9 F ) , blood pressure is 110/54 mm Hg , and pulse is 108 / min . The abdomen is diffusely tender with guarding and rebound tenderness . Bowel sounds are diminished . Abdominal imaging is shown above . Which of the following is the most likely diagnosis in this patient ? Bowel perforation Duodenal atresia Sigmoid volvulus Small bowel obstruction Toxic megacolon

Sigmoid volvulus This older patient with weeklong symptoms of vague lower left quadrant discomfort , nausea , and anorexia likely has diverticulitis . His presentation , now complicated by fever and peritonitis ( eg , guarding , rebound tenderness ) , is most consistent with bowel perforation . Because free ( ie , noncontained ) perforation causes colonic contents to spill into the peritoneal cavity , free air is typically visible on abdominal imaging ( eg , upright x - ray , CT scan ) . Patients with free perforation of the gastrointestinal tract in the setting of ongoing inflammation ( eg , diverticulitis ) often have a classic pain sequence : Moment of perforation : Sudden , severe pain ( + vomiting , syncope ) After perforation to 2 hours : Temporary relief or decreased pain as the inflamed organ decompresses > 2 hours after perforation : Generalized , constant pain due to peritonitis ( ± sepsis / septic shock ) Perforation of the bowel is an emergency that requires urgent surgical consult . (Choice B ) Duodenal atresia is a rare congenital obstruction that classically presents with the double bubble sign of air in the stomach and proximal duodenal pouch . It is principally identified shortly after birth , and affected infants generally have bilious emesis and absence of bowel movements ( ie , obstipation ) . (Choices C and E ) Abdominal radiographs in sigmoid volvulus frequently reveal a transition point at the sigmoid colon with an inverted U - shape dilation void of haustral markings and an absence of bowel gas distal to the torsion . Toxic megacolon is seen as a profound colonic dilation ( > 6 cm ) also with a loss of haustral folds . Perforation is a potential complication in both conditions , resulting in free air and generalized peritonitis only after bowel compromise . ( Choice D ) A small bowel obstru

Hyperkalemia

Tall T waves, prolonged PR interval, wide QRS

A 32 - year - old woman comes to the emergency department due to sudden - onset left flank pain and nausea . The pain radiates to the left groin and she is unable to find a comfortable position on the examination table . The pain is intermittent and waxes and wanes in severity . Temperature is 36.7 C ( 98 F ) , blood pressure is 140/90 mm Hg , and pulse is 92 / min . The patient has mild tenderness to percussion over the left flank . Bowel sounds are hypoactive . Which of the following recommendations would most likely prevent a recurrence of this patient's condition ? A. Avoid alcohol use B. Complete a course of antibiotics C. Drink plenty of water daily D. Follow a high - fiber diet E. Follow a low - calcium diet

This patient has acute flank pain and tenderness consistent with nephrolithiasis . The pain associated with nephrolithiasis ( renal colic ) is often severe and , although it may wax and wane , is generally not positional . The pain commonly radiates to the groin , especially as the stone passes down the ureter to the ureterovesical junction . Nausea and vomiting are common , and bowel sounds are often diminished due to an associated ileus . Hematuria is usually present but may not be grossly visible . Most kidney stones are calcium - based ( calcium oxalate , calcium phosphate ) . But regardless of chemical composition , low fluid intake can lead to supersaturation of urine with crystalline material and promote stone formation . Increasing fluid intake can reduce the risk of all types of stones . ( Choice A ) Excessive alcohol intake can trigger acute pancreatitis . The pain associated with pancreatitis is typically located in the epigastric area rather than the flank and radiates to the back rather than the groin . Choice B ) Recurrent infections of the upper urinary tract with urease - producing organisms ( eg , Klebsiella , Proteus ) can lead to formation of magnesium ammonium phosphate ( struvite ) stones . These stones are often large and may fill the renal pelvis . Although patients may have mild flank pain due to recurrent infection , acute renal colic is uncommon because these large stones do not travel down the ureter . ( Choice D ) A high - fiber diet is associated with a decreased risk of diverticulitis . This condition typically presents over a few days ( not suddenly , as in this patient ) with lower abdominal pain and tenderness in the left lower quadrant . Choice E ) Dietary calcium binds oxalate in the gut to form unabsorbable calcium oxalate . A low - calcium diet leads to increased absorption

A 45 - year - old woman comes to the emergency department for acute onset of low back pain radiating down her left leg after lifting heavy boxes at home . She has had no muscle weakness and has no concerns with bowel or bladder function . The back pain worsens with coughing or bearing down . Neurologic examination reveals 5/5 muscle strength in all 4 extremities with 2+ reflexes . Straight - leg raise sign is positive on the left . Sensory examination reveals reduced pinprick and temperature sensation along the posterolateral region of the left leg . X - ray of the lumbar spine reveals only degenerative changes . Which of the following is the best next step in management of this patient ? Bene density testing ( 1 % ) Gabapentin and tramadol ( 4 % ) Light exercise and a nonsteroidal anti - inflammatory drug ( 45 % ) MRI of the lumbar ( 36 % ) Physical therapy ( 12 % )

This patient's back pain that radiates down the leg and is worsened with coughing or bearing down is likely due to acute lumbosacral radiculopathy ( ALSR ) . ALSR is most often due to a mechanical compression of the L5 and / or S1 dorsal root ( eg , herniated disc ) . Patients typically have low back and leg pain ( sciatica ) , often with associated numbness and / or weakness in the leg . Reproduction of radicular pain with flexion at the hip and dorsiflexion at the ankle ( ie , positive straight - leg raise ) is relatively specific for the diagnosis . Most patients have spontaneous improvement and can be managed conservatively with nonsteroidal anti - inflammatory drugs ( NSAIDs ) and maintaining light - normal activity . A brief period of bed rest ( < 3 days ) is also acceptable , but extended bed rest has no benefit and may prolong recovery . If symptoms do not improve or there are new neurologic deficits at follow - up , MRI of the lumbosacral spine and surgical referral may be indicated . ( Choice A ) Bone density testing should be considered if there is evidence of compression fracture or sacral insufficiency fracture . This patient's x - ray reveals no evidence of fracture . ( Choice B ) Antispasmodics / muscle relaxants can be considered , but they are associated with significant sedation and inconsistent benefits . Opioids are less effective and are associated with sedation and a significant risk of dependence . Pain - modifying drugs such as gabapentin are primarily used for chronic symptoms and also cause significant sedation . ( Choice D ) Early MRI should be considered for patients with low back pain or ALSR who have " red flags , " such as a malignant or infectious etiology ( eg , fever , history of malignancy , intravenous drug abuse ) or significant neurologic deficit ( eg , saddle anesthesia , urina

DKA

This patient's presentation suggests diabetic ketoacidosis ( DKA ) from new - onset type 1 diabetes mellitus ( T1DM ) . DKA was likely triggered by her recent gastrointestinal illness ( ie , nausea and vomiting ) , which exacerbated dehydration and precipitated ketosis . Features of this patient's illness consistent with DKA include : • Young age : Although T1DM is classically associated with childhood onset , one - fourth of new cases are diagnosed in young adults . DKA is the second most common initial presentation of T1DM . Symptoms : Rapid onset of weight loss , polydipsia ( increased thirst ) , and diffuse abdominal pain occurs , possibly from acidosis and electrolyte abnormalities impairing gastrointestinal function ( eg , delayed gastric emptying , ileus Clinical signs : Dehydration ( evidenced by tachycardia and hypotension ) and hyperventilation ( ie , compensation for metabolic acidosis ) are prominent . Therefore , checking fingerstick glucose is the best next step in management . This test is rapid and minimally invasive ; it can . narrow the differential diagnosis and initial treatment for this patient's gastrointestinal symptoms and lethargy . Glucose levels of 300 800 mg / dL should prompt empiric treatment for DKA ( eg , insulin , intravenous fluids ) and confirmatory testing ( eg , serum ketones , arterial blood gas ) .

An 80 - year - old man is brought to the hospital due to increasing chest pain . Over the past 6 weeks , he has had dry cough , dysphagia , and a change in voice quality . The patient has a long history of hypertension . He is an ex - smoker with a 35 - pack - year history . On physical examination , the patient appears anxious , pale , and diaphoretic . Blood pressure is 80/60 mm Hg , pulse is 120 / min , and respirations are 20 / min . While being evaluated in the emergency department , he develops cardiac arrest . Which of the following is the most likely diagnosis in this patient ? A. Mallory - Weiss tear B. Pulmonary fat embolism C. Spontaneous pneumothorax . Thoracic aortic aneurysm E. Ventricular free - wall rupture

Thoracic aortic aneurysm

A 29 - year - old woman comes to the emergency department due to fever and headache for the last week . The patient has a generalized tonic - clonic seizure while being evaluated . Laboratory results are as follows : Hemoglobin 6.1 g / dL Platelets 16,000 / mm³ Creatinine 2.2 mg / dL Total bilirubin 4.3 mg / dL Serum haptoglobin undetectable PT 11 sec INR 1.1 Activated PTT 30 sec Peripheral blood smear shows schistocytes , as seen in the exhibit . Which of the following is the most likely diagnosis ? Acute myeloid leukemia Disseminated intravascular coagulation Glueese - 6 - phosphate dehydrogenase - deficiency Sickle cell anemia Thrombotic thrombocytopenic purpura

Thrombotic thrombocytopenic purpura

An 18 YO W is brought to the ER with severe vomiting, diarrhea, and dizziness that started this morning. 3 days ago, the pt had nasal bleeding that required anterior packing in the ER. Yesterday, she had pain and a sensation of pressure under her left eye and scant blood tinged nasal discharge. The pt has no sick contacts. T is 102 F, BP is 90/60 supine and 66/45 standing, P is 120, and RR are 23. She appears alert but restless. There is no nuchal rigidity. The pt has diffuse, confluent erythematous macules on her trunk and extremities. The oropharynx appears hyperemic. Lab results are: Platelets 55,000 Leukocytes 9,500 Bands 30 %. Which of the following is the most likely diagnosis in this patient ? A. Meningococcemia B. Toxic shock syndrome

Toxic shock syndrome

Acute diverticulitis Pathogenesis .

Trapped food particles & ↑ intracolonic pressure that cause microperforation of colonic diverticula

Hyperosmolar hyperglycemic state Patient characteristics

Type 2 diabetes mellitus . Older age

A 46 - year - old man comes to the office due to a 3 - day history of low back pain . The patient suddenly experienced moderately severe back pain while building a storage shed in his backyard . The pain shoots down the back of the leg into the foot and worsens during ambulation . He also has partial numbness in the left leg but no urinary or bowel symptoms . Vital signs are normal . With the patient supine on the table , raising the left leg beyond 45 degrees reproduces the pain . The ankle reflex is diminished on the left side compared to the right . MRI of the lumbosacral spine reveals S1 nerve root impingement . Which of the following is the most likely cause of this patient's nerve compression ? A. Large esteophyte at the facet joint Metastasis in the vertebral body Spinal epidural abscess Vertebral compression fracture Vertebral disc herniation

Vertebral disc herniation Educational objective Acute lumbosacral radiculopathy is most commonly caused by compression of the L5 or S1 nerve root due to vertebral disc herniation . Clinical findings include back pain radiating to the foot , diminished lower extremity reflexes , and a positive straight leg raise test .

A 32 - year - old man comes to the clinic due to a week of escalating low back pain . The pain is dull , achy , and present at rest and worsens with activity . He recently began using intravenous heroin due to significant psychosocial stress at home . Temperature is 36.7 C ( 98.1 F ) , blood pressure is 120/80 mm Hg , pulse is 90 / min , and respirations are 16 / min . Gentle percussion over the lumbar vertebrae ( L3 and L4 ) elicits exquisite pain . A full neurologic examination , including the straight - leg raising test , is normal . Which of the following is the most likely diagnosis in this patient ? A. Ankylosing spondylitis B. Lumbar dise herniation C. Lumbar spinal stenosis D. Vertebral compression fracture E. Vertebral osteomyelitis

Vertebral osteomyelitis This patient has vertebral osteomyelitis . Injection drug users , patients with sickle cell anemia , and immunosuppressed patients . are at highest risk for this condition . The spine is a frequent site of osteomyelitic infection in injection drug users . In this group , Staphylococcus aureus is the most common pathogen , but infections with gram - negative organisms also occur Most cases of vertebral osteomyelitis are chronic ( > 6 wk ) and insidious with minimal symptoms . Many patients have back pain unrelieved by rest ; fever is present in fewer than 50 % . Physical examination often shows few findings , but tenderness to gentle percussion over the spinous process of the involved vertebra can be an important clue . Leukocytes may be elevated or normal . The erythrocyte sedimentation rate ( ESR ) is often significantly elevated ( > 100 mm / hr ) . MRI is the most sensitive diagnostic study . Treatment is long - term intravenous antibiotics with or without surgery . ( Choice A ) Ankylosing spondylitis is characterized by pain and progressive limitation of back motion . It occurs most often in young men . Symptoms are usually most severe in the morning and improve as the day progresses . Exquisite tenderness to palpation is not a classic finding . Choice B ) Lumbar disc herniation typically presents with acute - onset back pain with or without radiation down the leg . Patients can usually recall an inciting event . The pain is worse with activity and improves with rest . The spine is not typically tender to palpation . ( Choice C ) Lumbar spinal stenosis is narrowing of the spinal canal with compres of one r more of the spinal roots . Most affected patients are age > 60 . Patients experience back pain that radiates to the buttocks and thighs . Symptoms often worsen with walking and lumbar ext

An 11 - year - old boy is brought to the urgent care clinic due to eye redness and discharge . The patient has had a sore throat for 4 days and right eye redness for 2 days . He says , " It sometimes feels like my eye is burning . " This morning , both eyes were crusted shut . His father used a moist towel to open the patient's eyelids and saw that the left eye was also red . The patient has had no eye pain , visual changes , or itching . Temperature is 37.4 C ( 99.3 F ) . Physical examination shows conjunctival injection and clear mucoid discharge in both eyes . Mild rhinorrhea and pharyngeal erythema are also present . Which of the following is the most likely diagnosis in this patient ? Allergic conjunctivitis Bacterial conjunctivitis Corneal abrasion Keratitis Viral conjunctivitis

Viral conjunctivitis Viral conjunctivitis is the most common cause of conjunctivitis in school - aged children . Presentation includes unilateral or bilateral eye redness and watery or mucoid discharge that can cause the eye to crust shut in the mornings . Most commonly due to adenovirus , viral conjunctivitis can present alone or in the setting of a viral prodrome ( eg , pharyngitis , rhinorrhea , fever ) , as in this patient . Patients often report a sandy or burning sensation in one or both eyes . The usual course of viral conjunctivitis is similar to that of viral upper respiratory tract infections : 3-5 days of worsening symptoms followed by gradual improvement and resolution in 1-2 weeks . Treatment is supportive with cool compresses and lubricating drops . Choice A ) Allergic conjunctivitis can also present with bilateral eye redness and watery discharge . However , this patient lacks the hallmark ocular itching ( pruritus ) found in allergic conjunctivitis . In addition , allergic conjunctivitis does not cause the gritty or burning sensation that this patient is experiencing . Choice B ) In contrast to viral conjunctivitis , bacterial conjunctivitis causes a more profuse , purulent discharge that accumulates soon . after being wiped away . Also , bacterial conjunctivitis generally presents in isolation rather than in the setting of other viral symptoms . Choices C and D ) Corneal abrasion and keratitis ( ie , corneal infection ) present with intense eye pain and photophobia , neither of which is present in this patient . Educational objective : Viral conjunctivitis is the most common cause of conjunctivitis in school - aged children . Presentation includes eye redness and mild watery or mucoid discharge , often in the setting of upper respiratory tract symptoms ( eg , pharyngitis , rhinorrhea ) .

A 57 - year - old woman comes to the emergency department after vomiting a substance that resembled " coffee grounds . " The patient now feels lightheaded and dizzy . Her medical history is significant for deep venous thrombosis , for which she takes warfarin . Blood pressure is 90/60 mm Hg , pulse is 110 / min , and respirations are 20 / min . Rectal examination shows maroon - colored , guaiac - positive stool . Which of the following is the best immediate treatment for this patient ? A. Cryoprecipitate B. Desmopressin C. Fresh frozen plasma ( FFP ) D. Platelet transfusion E. Protamine

Warfarin is commonly used for long - term anticoagulation in the setting of venous thrombosis and pulmonary thromboembolism . Warfarin inhibits vitamin K - dependent clotting factors , thereby causing an acquired coagulopathy that can lead to bleeding ( gastrointestinal hemorrhage with hematemesis and guaiac - positive stools , ecchymoses ) . To treat life - threatening bleeding in the setting of warfarin toxicity , rapid replenishment of clotting factors II , VII , IX , and X is generally performed through intravenous administration of fresh frozen plasma ( FFP ) , which contains all coagulation factors and other proteins present in the original unit of blood . Additional vitamin K should also be given . Vitamin K alone can be used in patients with abnormal coagulation tests ( eg , prolonged prothrombin or partial thromboplastin times ) but no bleeding . ( Choice A ) Cryoprecipitate ( precipitate obtained from FFP by centrifugation / thawing ) contains factor VIII , factor XIII , von Willebrand factor , and fibrinogen . Therefore , it would not be of significant benefit when vitamin K - dependent clotting factors ( II , VII , IX , and X ) require replenishment . ( Choice B ) Desmopressin increases plasma levels of von Willebrand factor and factor VIII ; it is used in management of hemophilia A and von Willebrand disease . Choice D ) Warfarin toxicity does not affect platelet count ; therefore , a platelet transfusion i s not indicated . ( Choice E ) Protamine sulfate is used to treat heparin overdose

A 53 - year - old woman who was recently treated with amoxicillin for an ear infection follows up for an itchy rash . The patient reports that she has completed the course of amoxicillin with resolution of ear pain , ear pressure , and muffled hearing . She has no history of renal disease , hypertension , or allergic reactions . Temperature is 38.1 C ( 100.6 F ) , blood pressure is 128/82 mm Hg , and pulse is 86 min . Physical examination shows a diffuse , erythematous maculopapular rash on the trunk and upper extremities . Laboratory evaluation shows serum creatinine of 2.1 mg / dL and eosinophilia . Which of the following is the most likely finding on urine studies ? Oval fat bodies Positive leukeeyte esterase and nitrite Protein exeretion > 3 g / day Red blood cell casts White blood cell casts

White blood cell casts Acute interstitial nephritis Causes Medications ( antibiotics , NSAIDs , PPIs ) Infections ( eg , Legionella , Streptococcus ) Systemic / autoimmune disease ( eg , lupus , sarcoidosis )

A 26 - year - old man presents with right ankle pain . The patient was playing basketball when he jumped in the air , landed , rolled his foot in medially , and immediately fell to the ground . He tried to walk off the basketball court but was unable bear weight on the foot . Vital signs are normal . The lateral aspect of the right ankle has significant swelling and tenderness over the lateral malleolus . There is no bruising and no tenderness over the anterior aspect of the foot . The patient can plantar flex and dorsiflex the ankle . Sensation is intact . Which of the following is the best next step in management of this patient ? Ankle splinting and nonsteroidal anti - inflammatory drugs Immediate surgical fixation Intraarticular aspiration MRI of the ankle and foot X - ray of the ankle

X - ray of the ankle

Ventricular Tachycardia treatment

cardiac ( wide Educational Immediate clinical

Jones Fracture

fracture of the base of the 5th metatarsal

Anterior uveitis

inflammation of the iris and ciliary body

Hypercalcemia

prolonged QT interval

Treatment of VTach

this patient is hemodynamically stable ; signs of hemodynamic instability include ischemic chest pain , severe hypotension / shock , pulmonary edema , and confusion . Patients with wide - complex tachycardia who are hemodynamically unstable should undergo immediate synchronized cardioversion . In contrast , hemodynamically stable patients , such as this one , can undergo attempted pharmacologic cardioversion with an antiarrhythmic agent . Amiodarone , a class III antiarrhythmic that primarily inhibits potassium channels and slows ventricular repolarization , is commonly considered the first - line pharmacologic therapy for monomorphic VT and may restore normal sinus rhythm . Other options include procainamide , lidocaine , and sotalol Once VT is stabilized , further evaluation is indicated to determine its underlying cause . Possible etiologies and / or contributing factors include current or previous myocardial infarction ( eg , ischemic scarring electrolyte abnormalities , hypoxemia , adverse drug effects , and heart failure with reduced ejection fraction . ( Choice B ) Atropine is the initial treatment of choice for patients with hemodynamically unstable sinus bradycardia . Atropine increases the heart rate ( and cardiac output ) by decreasing parasympathetic tone at the sinoatrial node , so it should be avoided in patients with tachycardia . ( Choice C ) Digoxin is sometimes used in the treatment of supraventricular tachyarrhythmias such as atrial fibrillation , but it is not indicated for the treatment of VT . ( Choices D and E ) Metoprolol and verapamil may be used to treat supraventricular tachycardia if adenosine is ineffective in terminating the rhythm . These medications reduce ventricular automaticity and may be useful in preventing recurrences of VT ; however , neither one is typically useful in the p

Acute diverticulitis Clinical presentation

• Abdominal pain ( usually left lower quadrant ) • Nausea , vomiting , change in bowel habits • Tenderness t palpable mass • Leukocytosis

Pemphigus vulgaris Clinical Features

• Age 30-40 Autoimmune disorder • Painful , flaccid bullae features Mucosal erosions Nikolsky sign : skin detachment with pressure

Hyperosmolar hyperglycemic state Initial management

• Aggressive hydration with normal saline * Intravenous insulin Potassium replacement even when serum levels are high - normal ( eg , < 5.3 mEq / L )

Guillain - Barré syndrome Diagnosis

• Clinical • Supportive findings Cerebrospinal fluid : ↑ protein , normal leukocytes o Abnormal electromyography & nerve conduction o MRI : normal or enhancement of anterior nerve roots / cauda equina

Complications of Mastoiditis

• Extracranial extension ( subperiosteal abscess , facial nerve palsy , hearing loss , labyrinthitis ) Intracranial extension ( brain abscess , meningitis )

Clinical findings of Mastoiditis

• Fever & otalgia Inflammation of mastoid Protrusion of the auricle Opacification of mastoid air cells on CT scan or MRI

Hyperosmolar hyperglycemic state Laboratory studies

• Glucose > 600 mg / dL ( frequently > 1,000 mg / dL ) • Normal pH & bicarbonate . Normal anion gap • Negative or small serum ketones Serum osmolality > 320 mOsm / kg

Hyperosmolar hyperglycemic state Clinical symptoms

• Gradual hyperglycemic symptoms ( eg , polyuria , polydipsia ) Altered mentation ( eg , coma , focal signs , seizures

Mastoiditis Patho

• Infection of the mastoid air cells . Complication of acute otitis media . Most commonly due to Streptococcus pneumoniae

Management of Mastoiditis

• Intravenous antibiotics • Drainage of purulent material ( eg , tympanostomy , mastoidectomy )

Traumatic hyphema Initial management

• Ophthalmology consultation o Monitor intraocular pressure • Cycloplegic & glucocorticoid eye drops • Eye shield Bed rest ( elevate head )


Conjuntos de estudio relacionados

Is the sentence simple, compound, complex, or compound-complex? IXL

View Set

K Ch. 5: Foundations of Employee Motivation

View Set

CISSP Chapter 6: Cryptography and Symmetric Key Algorithms

View Set

ASTRO Module 6: Capturing Light-Technology of the Telescope

View Set